Как найти доказательства треугольника

Треугольники. Признаки равенства треугольников

Треугольник − это геометрическая фигура, образованная соединением отрезками трех, не лежащих на одной прямой точек .

Эти точки называются вершинами треугольника. Отрезки, соединяющие эти точки называются сторонами треугольника.

Треугольник обозначается знаком ⊿. Например треугольник ABC обозначается так: ⊿ABC. Этот же треугольник можно обозначать так: ⊿BAC, ⊿CBA и т.д.

Углы треугольника обозначают так ∠BAC, ∠ABC, ∠BCA. Эти же углы коротко обозначают также ∠A, ∠B, ∠C, соответственно. Углы треугольника принято также обозначать греческими буквами α, β, γ и т.д. Стороны тркеугольника обозначают так AB, BC, AC. Принято также стороны обозначать одной строчной буквой, причем сторона напротив угла A ,обозначается буквой a, сторона напротив угла Bb, сторона напротив угла Cc. Сумма трех сторон треугольника называется периметром треугольника.

Как известно, две треугольники называются равными, если при наложении друг на друга их можно совместить. На Рис.2 представлены два треугольника ABC и A1B1C1. Треугольник ABC можно наложить на треугольник A1B1C1 так, чтобы вершины и стороны этих треугольников попарно совместились. Очевидно, что при этом совместятся и соответствующие углы.

Вышеизложенное можно сформулировать так:

Если два треугольника равны, то элементы (стороны и углы) одного треугольника соответственно равны элементам другого треугольника. Равенство треугольников ABC и A1B1C1 обозначается так:

Первый признак равенства треугольников

Теорема 1. Если две стороны и угол между ними одного треугольника соответственно равны двум сторонам и углу между ними другого треугольника, то эти треугольники равны.

Доказательство. Рассмотрим треугольники ABC и A1B1C1 (Рис.3). Пусть AB=A1B1, =A1С1 и ∠A=∠A1. Докажем, что .

Второй признак равенства треугольников

Теорема 2. Если сторона и два прилежащих к ней угла одного треугольника соответственно равны стороне и двум прилежащим к ней углам другого треугольника, то эти треугольники равны.

Доказательство. Рассмотрим треугольники ABC и A1B1С1 (Рис.4). Пусть AB=A1B1, ∠A=∠A1, ∠B=∠B1. Докажем, что .

Третий признак равенства треугольников

Теорема 3. Если три стороны одного треугольника соответственно равны трем сторонам другого треугольника, то эти треугольники равны.

Доказательство. Рассмотрим треугольники ABC и A1B1С1. Пусть AB=A1B1, AC=A1C1 и BC=B1C1. Докажем, что . Приложим треугольник ABC к треугольнику A1B1С1 так, чтобы вершина A совмещалась с вершиной A1, вершина B совмещалась с вершиной B1, а вершины С и С1 находились по разные стороны от прямой A1B1.

Возможны три варианта: луч CC1 проходит внутри угла ACB(Рис.6); луч CC1 совпадает с одной из сторон угла ACB (Рис.7); луч CC1 проходит вне угла ACB(Рис.8). Рассмотрим эти три случая по отдельности.

Имеем AC=A1C1, BC=B1C1ACB=∠A1C1B1 и по первому признаку равенства треугольников . Теорема доказана.

Вариант 2 (Рис.7). Так как по условию теоремы AC=A1C1 и BC=B1C1, то треугольник BСС1 равнобедренный. Тогда ∠1=∠2. Имеем: AC=A1C1, BC=B1C1, ∠1=∠2 и по первому признаку равенства треугольников . Теорема доказана.

Вариант 3 (Рис.8). Так как по условию теоремы AC=A1C1 и BC=B1C1, то треугольники AСС1 и BСС1 равнобедренные. Тогда ∠1=∠2 и и, следовательно:

Имеем AC=A1C1, BC=B1C1 и по первому признаку равенства треугольников . Теорема доказана.

Задачи и решения

Задача 1. На сторонах угла CAD отмечены точки B и E так, что точка B лежит на отрезке AC, а точка E − на отрезке AD, причем AC=AD и AB=AE. Докажите, что ∠CBD=∠DEC (Рис.9).

Доказательство. AC=AD, AE=AB, ∠CAD общий для треугольников CAE и DAB. Тогда, по первому признаку равенства треугольников (теорема 1) ⊿ACE=⊿ADB. Следовательно ∠DBA=∠AEC. Поскольку углы CBD и DBA смежные, то CBD=180°−∠DBA. Аналогично CED=180°-∠AEC. То есть ∠CBD=∠DEC. Конец доказательства .

Задача 2. По данным рисунка рис.10 докажите, что OP=OT, ∠P=∠T

Доказательство. OC=OB, ∠TCO=∠PBO=90°. Углы TOC и POB вертикальные (следовательно равны) тогда, повторому признаку равенства треугольников (теорема 2), ⊿TCO=⊿PBO. Конец доказательства .

Признаки равенства треугольников

О чем эта статья:

Первый признак равенства треугольников

Конечно, равенство треугольников всегда можно доказать наложением одного треугольника на другой. Но, согласитесь, — это несерьезно. Какое может быть наложение, когда есть три теоремы и можно их доказать.

Давайте рассмотрим три признака равенства треугольников.

Теорема 1. Равенство треугольников по двум сторонам и углу между ними.

Если две стороны и угол между ними одного треугольника соответственно равны двум сторонам и углу между ними другого треугольника, то такие треугольники равны.

При наложении △A1B1C1 на △ABC вершина A1 совмещается с вершиной A, и сторона A1B1 накладывается на сторону AB, AC — на сторону A1C1.

Сторона A1B1 совмещается со стороной AB, вершина B совпадает с вершиной B1, сторона A1С1 совмещается со стороной AС, вершина C совпадает с вершиной C1.

Значит, происходит совмещение вершин В и В1, С и С1.

Второй признак равенства треугольников

Теорема 2. Равенство треугольников по стороне и двум прилежащим к ней углам.

Если сторона и два прилежащих к ней угла одного треугольника соответственно равны стороне и двум прилежащим к ней углам другого треугольника, то такие треугольники равны.

Путем наложения △ABC на △A1B1C1, совмещаем вершину А с вершиной A1, вершины В и В1 лежат по одну сторону от А1С1.

Тогда АС совмещается с A1C1, вершина C совпадает с C1, поскольку мы знаем, что АС = A1C1.

AB накладывается на A1B1, поскольку мы знаем, что ∠A = ∠A1.

CB накладывается на C1B1, поскольку мы знаем, что ∠C = ∠C1.

Вершина B совпадает с вершиной B1.

Третий признак равенства треугольников

Теорема 3. Равенство треугольников по трем сторонам.

Если три стороны одного треугольника соответственно равны трем сторонам другого треугольника, то такие треугольники равны.

Доказательство 3 признака равенства треугольников:

Приложим △ABC к △A1B1C1 таким образом, чтобы вершина A совпала с вершиной A1, вершина B — с вершиной B1, вершина C и вершина C1 лежат по разные стороны от прямой А1В1.

Кроме трех основных теорем, запомните еще несколько признаков равенства треугольников.

Равны ли треугольники, можно определить не только по сторонам и углам, но и по высоте, медиане и биссектрисе.

  1. Если угол, сторона, противолежащая этому углу, и высота, опущенная на другую сторону, одного треугольника соответственно равны углу, стороне и высоте другого треугольника — такие треугольники равны.
  2. Если две стороны и медиана, заключенная между ними, одного треугольника соответственно равны двум сторонам и медиане другого треугольника — такие треугольники равны.
  3. Если сторона и две медианы, проведенные к двум другим сторонам, одного треугольника соответственно равны стороне и двум медианам другого треугольника — такие треугольники тоже равны.
  4. Если две стороны и биссектриса, заключенная между ними, одного треугольника соответственно равны двум сторонам и биссектрисе другого треугольника — вы уже догадались сами: эти ребята равны.
  5. Два треугольника равны, если сторона, медиана и высота, проведенные к другой стороне, одного треугольника соответственно равны стороне, медиане и высоте другого треугольника.

Как видите, доказать равенство треугольников можно по множеству признаков и десятком способов. Три признака равенства треугольников — основные. Все остальные способы также стоит запомнить, ведь треугольник — только с виду простая фигура.

Как установить и доказать, что треугольники равны

Геометрия как отдельный предмет начинается у школьников в 7 классе. До этого времени они касаются геометрических задач достаточно лёгкой формы и в основном того, что можно рассмотреть на наглядных примерах: площади комнаты, земельного участка, длины и высоты стен в помещениях, периметра плоских предметов и прочее. В нача ле изучения непосредственно геометрии появляются первые сложности, такие, например, как понятие прямой, так как потрогать руками эту прямую нет возможности. Что касается треугольников -это самый простой вид многоугольников, содержащий всего три угла и три стороны.

Тема треугольников одна из основных важных и больших тем школьной программы в геометрии 7−9 классов. Усвоив её хорошо, возможно решать очень сложные задачи. При этом можно изначально рассматривать совершенно другую геометрическую фигуру, а затем разделить её для удобства на подходящие треугольные части.

Как доказать, что треугольники равны

Для работы над доказательством равенства ∆ ABC и ∆A1B1C1 нужно хорошо усвоить признаки равенства фигур и уметь ими пользоваться. Перед изучением признаков необходимо научиться определять равенство сторон и углов простейших многоугольников.

Чтобы доказать, что углы треугольников равны, помогут следующие варианты:

  1. ∠ α = ∠ β исходя из построения фигур.
  2. Дано в условии задания.
  3. При двух параллельных прямых и наличии секущей могут образоваться как внутренние накрест лежащие, так и соответственные ∠ α = ∠ β.
  4. Прибавляя (вычитая) к (из) ∠ α = ∠ β равные углы.
  5. Всегда сходны вертикальные ∠ α и ∠ β
  6. Общий ∠ α, одновременно принадлежащий ∆ MNK и ∆ MNH .
  7. Биссектриса делит ∠ α на два равнозначных.
  8. Смежный с 90° — угол, равный исходному.
  9. Смежные равным углам равны.
  10. Высота образует два смежных 90° .
  11. В равнобедренном ∆ MNK при основании ∠ α = ∠ β.
  12. В равных ∆ MNK и ∆ SDH соответствующие ∠ α = ∠ β.
  13. Доказанное ранее равенство ∆ MNK и ∆ SDH .

Это интересно: Как найти периметр треугольника.

3 признака равенства треугольников

Доказательство равенства ∆ ABC и ∆A1B1C1 очень удобно производить, опираясь на основные признаки тождественности этих простейших многоугольников. Существует три таких признака. Они являются очень важными при решении многих геометрических задач. Стоит рассмотреть каждый.

  • I признак. Две стороны и угол между ними ∆ ABC соответственно = двум сторонам и углу ∆ A1B1C1 , следовательно, треугольники равны.
  • II признак. Сторона и два прилежащих к ней угла ∆ ABC соответственно = стороне и двум углам ∆ A1B1C1 , => треугольники равны.
  • III признак. Если все стороны ∆ ABC соответственно = сторонам ∆ A1B1C1 , то имеющиеся треугольники равны.

Перечисленные выше признаки являются теоремами и доказываются методом наложения одной фигуры на другую, соединения вершин соответственных углов и начала лучей. Доказательства равенства треугольников в 7 классе описаны в очень доступной форме, но сложны в изучении школьниками на практике, так как содержат большое количество элементов, обозначенных заглавными латинскими буквами. Это не совсем привычно для многих учеников на момент начала изучения предмета. Подростки путаются в названиях сторон, лучей, углов.

Доказательство подобия треугольников

Чуть позже появляется ещё одна важная тема «Подобие треугольников». Само определение «подобие» в геометрии означает схожесть формы при различии размеров. Для примера можно взять два квадрата, первый со стороной 4 см, а второй 10 см. Эти виды четырёхугольников будут похожи и, одновременно, иметь отличие, поскольку второй будет больше, причём каждая сторона увеличена в одинаковое количество раз.

В рассмотрении темы подобия также приводятся 3 признака:

  • Первый — о двух соответственно равных углах двух рассматриваемых треугольных фигур.
  • Второй — об угле и образующих его сторонах ∆ MNK , которые равны соответственным элементам ∆ SDH .
  • Третий — указывает на пропорциональность всех соответственных сторон двух нужных фигур.

Как же доказать, что треугольники подобны? Достаточно воспользоваться одним из выше перечисленных признаков и грамотно описать весь процесс доказательства задания. Тема подобия ∆ MNK и ∆ SDH проще воспринимается школьниками исходя из того, что к моменту её изучения ученики уже свободно пользуются обозначениями элементов в геометрических построениях, не путаются в огромном количестве названий и умеют читать чертежи.

Завершая прохождение обширной темы треугольных геометрических фигур, учащиеся уже в совершенстве должны знать, как доказать равенство ∆ MNK = ∆ SDH по двум сторонам, установить равны два треугольника или нет. Учитывая, что многоугольник, имеющий ровно три угла — это одна из важнейших геометрических фигур, к усвоению материала следует подойти серьёзно, уделяя особое внимание даже мелким фактам теории.

источники:

http://skysmart.ru/articles/mathematic/priznaki-ravenstva-treugolnika

Как установить и доказать, что треугольники равны

Треугольники. Признаки равенства треугольников

Треугольник − это геометрическая фигура, образованная соединением отрезками трех, не лежащих на одной прямой точек .

Эти точки называются вершинами треугольника. Отрезки, соединяющие эти точки называются сторонами треугольника.

Треугольник обозначается знаком ⊿. Например треугольник ABC обозначается так: ⊿ABC. Этот же треугольник можно обозначать так: ⊿BAC, ⊿CBA и т.д.

Углы треугольника обозначают так ∠BAC, ∠ABC, ∠BCA. Эти же углы коротко обозначают также ∠A, ∠B, ∠C, соответственно. Углы треугольника принято также обозначать греческими буквами α, β, γ и т.д. Стороны тркеугольника обозначают так AB, BC, AC. Принято также стороны обозначать одной строчной буквой, причем сторона напротив угла A ,обозначается буквой a, сторона напротив угла Bb, сторона напротив угла Cc. Сумма трех сторон треугольника называется периметром треугольника.

Как известно, две треугольники называются равными, если при наложении друг на друга их можно совместить. На Рис.2 представлены два треугольника ABC и A1B1C1. Треугольник ABC можно наложить на треугольник A1B1C1 так, чтобы вершины и стороны этих треугольников попарно совместились. Очевидно, что при этом совместятся и соответствующие углы.

Вышеизложенное можно сформулировать так:

Если два треугольника равны, то элементы (стороны и углы) одного треугольника соответственно равны элементам другого треугольника. Равенство треугольников ABC и A1B1C1 обозначается так:

Первый признак равенства треугольников

Теорема 1. Если две стороны и угол между ними одного треугольника соответственно равны двум сторонам и углу между ними другого треугольника, то эти треугольники равны.

Доказательство. Рассмотрим треугольники ABC и A1B1C1 (Рис.3). Пусть AB=A1B1, =A1С1 и ∠A=∠A1. Докажем, что .

Так как ∠A=∠A1, то треугольник ABC можно наложить на треугольник A1B1C1 так, чтобы вершины A и A1 совпадали, а стороны AB и наложились на лучи A1B1 и A1C1, соответственно.

Так как по условию теоремы AB=A1B1, =A1С1, то сторона AB совместится со стороной A1B1, а сторона − со стороной A1С1.Тогда совместятся B и B1, C и С1. Следовательно сторона BC совместится со стороной B1C1. То есть треугольники ABC и A1B1C1 полностью совместятся. Теорема доказана.

Второй признак равенства треугольников

Теорема 2. Если сторона и два прилежащих к ней угла одного треугольника соответственно равны стороне и двум прилежащим к ней углам другого треугольника, то эти треугольники равны.

Доказательство. Рассмотрим треугольники ABC и A1B1С1 (Рис.4). Пусть AB=A1B1, ∠A=∠A1, ∠B=∠B1. Докажем, что .

Наложим треугольник ABC на треугольник A1B1С1 так, чтобы вершина A совмещалась с вершиной A1, сторона AB − со стороной A1B1 (по условию теоремы AB=A1B1), а вершины C и С1 оказались по одну сторону от прямой A1B1.

Так как ∠A=∠A1 и ∠B=∠B1, то сторона наложится на луч A1C1 а сторона − на луч B1С1. Тогда вершина C окажется на луче A1C1 и на луче B1C1. Т.е. она окажется на пересечении этих лучей и, следовательно, вершина C совместится с общей точкой лучей A1C1 и B1C1, т.е. с вершиной C1. Таким образом совместятся стороны AC и A1C1, BC и B1C1. То есть треугольники ABC и A1B1С1 полностью совместятся, поэтому они равны. Теорема доказана.

Третий признак равенства треугольников

Теорема 3. Если три стороны одного треугольника соответственно равны трем сторонам другого треугольника, то эти треугольники равны.

Доказательство. Рассмотрим треугольники ABC и A1B1С1. Пусть AB=A1B1, AC=A1C1 и BC=B1C1. Докажем, что . Приложим треугольник ABC к треугольнику A1B1С1 так, чтобы вершина A совмещалась с вершиной A1, вершина B совмещалась с вершиной B1, а вершины С и С1 находились по разные стороны от прямой A1B1.

Возможны три варианта: луч CC1 проходит внутри угла ACB(Рис.6); луч CC1 совпадает с одной из сторон угла ACB (Рис.7); луч CC1 проходит вне угла ACB(Рис.8). Рассмотрим эти три случая по отдельности.

Вариант 1 (Рис.6). Так как по условию теоремы AC=A1C1 и BC=B1C1, то треугольники AСС1 и BСС1 равнобедренные. Тогда ∠1=∠2 и ∠3=∠4 и, следовательно:

Имеем AC=A1C1, BC=B1C1ACB=∠A1C1B1 и по первому признаку равенства треугольников . Теорема доказана.

Вариант 2 (Рис.7). Так как по условию теоремы AC=A1C1 и BC=B1C1, то треугольник BСС1 равнобедренный. Тогда ∠1=∠2. Имеем: AC=A1C1, BC=B1C1, ∠1=∠2 и по первому признаку равенства треугольников . Теорема доказана.

Вариант 3 (Рис.8). Так как по условию теоремы AC=A1C1 и BC=B1C1, то треугольники AСС1 и BСС1 равнобедренные. Тогда ∠1=∠2 и и, следовательно:

Имеем AC=A1C1, BC=B1C1 и по первому признаку равенства треугольников . Теорема доказана.

Задачи и решения

Задача 1. На сторонах угла CAD отмечены точки B и E так, что точка B лежит на отрезке AC, а точка E − на отрезке AD, причем AC=AD и AB=AE. Докажите, что ∠CBD=∠DEC (Рис.9).

Доказательство. AC=AD, AE=AB, ∠CAD общий для треугольников CAE и DAB. Тогда, по первому признаку равенства треугольников (теорема 1) ⊿ACE=⊿ADB. Следовательно ∠DBA=∠AEC. Поскольку углы CBD и DBA смежные, то CBD=180°−∠DBA. Аналогично CED=180°-∠AEC. То есть ∠CBD=∠DEC. Конец доказательства.

Задача 2. По данным рисунка рис.10 докажите, что OP=OT, ∠P=∠T

Доказательство. OC=OB, ∠TCO=∠PBO=90°. Углы TOC и POB вертикальные (следовательно равны) тогда, повторому признаку равенства треугольников (теорема 2), ⊿TCO=⊿PBO. Конец доказательства.

Содержание:

Если на плоскости отметить три точки А, В и С, не лежащие на одной прямой, и соединить их отрезками, то получим треугольник ABC. Можно сказать, что треугольник — это трехзвенная замкнутая ломаная. Обозначают: Признаки равенства треугольников - определение и вычисление с примерами решения

Определения

Признаки равенства треугольников - определение и вычисление с примерами решения

Определение. Треугольником называется трехзвенная замкнутая ломаная вместе с частью плоскости, которую она ограничивает.

Если соединить концами три деревянных планки, то получится треугольник, который нельзя подвергнуть деформации — он будет сохранять свою форму. Тогда как четырехугольник может менять свою форму (рис. 102)? Это свойство «жесткости» треугольника широко используется в технике, производстве, строительстве.
Признаки равенства треугольников - определение и вычисление с примерами решения

Равные треугольники

Равные треугольники можно совместить наложением так, что соответственно совпадут все три стороны и все три угла (рис. 103). В совпавших, то есть в равных треугольниках, против равных сторон лежат равные углы, а против равных углов — равные стороны. Если Признаки равенства треугольников - определение и вычисление с примерами решения то Признаки равенства треугольников - определение и вычисление с примерами решения а если Признаки равенства треугольников - определение и вычисление с примерами решения то Признаки равенства треугольников - определение и вычисление с примерами решения

Признаки равенства треугольников - определение и вычисление с примерами решения

Для совмещения равных отрезков достаточно совпадения их концов, а для совмещения равных треугольников — совпадения их вершин.

Виды треугольников

Если у треугольника все три стороны имеют разную длину, то такой треугольник называется разносторонним.

Треугольник, у которого две стороны равны, называется равнобедренным. Его равные стороны называются боковыми сторонами, третья сторона — основанием, вершина, противолежащая основанию, — вершиной равнобедренного треугольника (рис. 104).

Признаки равенства треугольников - определение и вычисление с примерами решения

Если у треугольника равны все три стороны, то он называется равносторонним (рис. 105). Равносторонний треугольник является также и равнобедренным, где любую пару сторон можно принять за боковые стороны.

Признаки равенства треугольников - определение и вычисление с примерами решения

По величине углов треугольники делятся на остроугольные (у них все углы острые), тупоугольные (есть тупой угол) и прямоугольные (есть прямой угол) (рис. 106).

Признаки равенства треугольников - определение и вычисление с примерами решения

Подведем итоги.

Треугольником называется трехзвенная замкнутая ломаная вместе с частью плоскости, которую она ограничивает.

Периметром треугольника (многоугольника) называется сумма длин его сторон.

Равными треугольниками называются треугольники, которые можно совместить наложением.

Равнобедренным треугольником называется треугольник, у которого две стороны равны.

Равносторонним треугольником называется треугольник, у которого все стороны равны.

Свойство равных треугольников. В равных треугольниках против равных сторон лежат равные углы, а против равных углов — равные стороны.

Замечание. Называя или записывая равные треугольники, стараются соблюдать последовательность соответствующих вершин. Во многих случаях это удобно. Однако делать это необязательно. Обе записи: Признаки равенства треугольников - определение и вычисление с примерами решенияАВС =Признаки равенства треугольников - определение и вычисление с примерами решенияKNM и Признаки равенства треугольников - определение и вычисление с примерами решенияBAC =Признаки равенства треугольников - определение и вычисление с примерами решенияKNM — правильные. Иногда соответствующие вершины равных треугольников обозначают одними и теми же буквами, добавляя к буквам одного из треугольников индекс: Признаки равенства треугольников - определение и вычисление с примерами решенияАВС = = Признаки равенства треугольников - определение и вычисление с примерами решенияА1В1С1. При такой записи имеют в виду, что соответствующими являются вершины А и А1, В и В1, С и С1.

Первый и второй признаки равенства треугольников

При выяснении равны ли треугольники нет необходимости устанавливать равенство всех их соответствующих элементов путем наложения или измерения. Следующие две теоремы гарантируют равенство треугольников при равенстве некоторых сторон и углов.

Теорема (первый признак равенства треугольников). Если две стороны и угол между ними одного треугольника соответственно равны двум сторонам и углу между ними другого треугольника, то такие треугольники равны.

Дано: АВ =А1В1, АС =А1С1, Признаки равенства треугольников - определение и вычисление с примерами решенияA = Признаки равенства треугольников - определение и вычисление с примерами решенияA1 (рис. 108).

Признаки равенства треугольников - определение и вычисление с примерами решения

Доказать: Признаки равенства треугольников - определение и вычисление с примерами решенияАВС = Признаки равенства треугольников - определение и вычисление с примерами решенияА1В1С1.

Доказательство:

Наложим треугольник ABC на треугольник А1В1С1 так, чтобы совпали равные углы А и А1, луч АВ совпал с лучом А1В1, а луч АС совпал с лучом А1С1. Так как отрезки АВ и А1В1 равны, то они совпадут при наложении, и вершина В совпадет с вершиной В1. Аналогично совпадут равные отрезки АС и A1C1, вершина С совпадет с вершиной C1. Треугольники совпадут полностью, так как совпадут их вершины. Таким образом, Признаки равенства треугольников - определение и вычисление с примерами решенияАВС = Признаки равенства треугольников - определение и вычисление с примерами решенияА1В1С1. Теорема доказана.

Говорят, что две стороны и угол между ними задают треугольник однозначно.

Теорема (второй признак равенства треугольников). Если сторона и два прилежащих к ней угла одного треугольника соответственно равны стороне и двум прилежащим к ней углам другого треугольника, то такие треугольники равны.

Дано:

AC =А1С1, Признаки равенства треугольников - определение и вычисление с примерами решенияA = Признаки равенства треугольников - определение и вычисление с примерами решенияА1, Признаки равенства треугольников - определение и вычисление с примерами решенияC = Признаки равенства треугольников - определение и вычисление с примерами решенияС1 (рис. 109).

Доказать: Признаки равенства треугольников - определение и вычисление с примерами решенияАВС = Признаки равенства треугольников - определение и вычисление с примерами решенияА1В1С1.

Доказательство:

Наложим треугольник ABC на треугольник А1В1С1 так, чтобы совпали равные стороны АС и А1С1, угол А совпал с равным углом А1, а угол С — с равным углом Сх. Тогда луч АВ совпадет с лучом А1В1, луч СВ — с лучом С1В1, а вершина В совпадет с вершиной В1 (точка В будет принадлежать и прямой
А1В1, и прямой С1В1, и поэтому совпадет с точкой их пересечения В1). Треугольники совпадут полностью, так как совпадут их вершины. Таким образом, Признаки равенства треугольников - определение и вычисление с примерами решенияАВС = Признаки равенства треугольников - определение и вычисление с примерами решенияА1В1С1. Теорема доказана.

Говорят, что сторона и два прилежащих к ней угла задают треугольник однозначно

Пример №1

Отрезки АВ и CD пересекаются в их серединах. Доказать, что расстояния между точками А и С, В и D равны.

Признаки равенства треугольников - определение и вычисление с примерами решения

Доказательство:

Пусть О — точка пересечения отрезков АВ и CD (рис. 110). Рассмотрим Признаки равенства треугольников - определение и вычисление с примерами решенияАОС и Признаки равенства треугольников - определение и вычисление с примерами решенияBOD. У них АО = ОВ, CO = OD по условию, Признаки равенства треугольников - определение и вычисление с примерами решенияAOC = Признаки равенства треугольников - определение и вычисление с примерами решенияBOD как вертикальные. Треугольники равны по двум сторонам и углу между ними, то есть по 1-му признаку равенства треугольников. Стороны АС и BD равны, так как в равных треугольниках против равных углов лежат равные стороны.

Возможно краткое оформление решения задачи.Признаки равенства треугольников - определение и вычисление с примерами решения

Пример №2

Дана простая замкнутая ломаная ABCD, у которой АВ =AD = 6 см, CD -4 см и луч АС является биссектрисой угла BAD. Найти длину ломаной ABCD.

Решение:

У треугольников ABC и ADC сторона АС — общая (рис. 111), AB=AD по условию, Признаки равенства треугольников - определение и вычисление с примерами решенияBAC =Признаки равенства треугольников - определение и вычисление с примерами решенияDAC, так как АС — биссектриса угла BAD.

Признаки равенства треугольников - определение и вычисление с примерами решения

Эти треугольники равны по 1-му признаку равенства треугольников.

Отсюда ВС = CD как соответствующие (соответственные) стороны в двух равных треугольниках.

Длина ломаной ABCD: Признаки равенства треугольников - определение и вычисление с примерами решения

Ответ: 20 см.

Пример №3

На сторонах угла В отложены отрезки: ВА = ВС, КА-МС (рис. 112). Доказать, что Признаки равенства треугольников - определение и вычисление с примерами решенияA = Признаки равенства треугольников - определение и вычисление с примерами решенияС.

Признаки равенства треугольников - определение и вычисление с примерами решения

Доказательство:

Рассмотрим треугольники АВМ и СВК. У них Признаки равенства треугольников - определение и вычисление с примерами решенияB — общий, АВ = СВ по условию, MB=KB, так как MB = СВ — СМ, KB =АВ -АК (если от равных отрезков отнять равные, получим равные отрезки). Треугольники АВМ и СВК равны по двум сторонам и углу между ними. Из равенства треугольников следует, что Признаки равенства треугольников - определение и вычисление с примерами решенияA = Признаки равенства треугольников - определение и вычисление с примерами решенияC (в равных треугольниках против равных сторон лежат равные углы).

Пример №4

На рисунке 113 Признаки равенства треугольников - определение и вычисление с примерами решенияBAD = Признаки равенства треугольников - определение и вычисление с примерами решенияCDA, Признаки равенства треугольников - определение и вычисление с примерами решенияCAD = Признаки равенства треугольников - определение и вычисление с примерами решенияBDA. Доказать равенство треугольников АОВ и DOC.

Признаки равенства треугольников - определение и вычисление с примерами решения

Доказательство:

Так как Признаки равенства треугольников - определение и вычисление с примерами решенияABD =Признаки равенства треугольников - определение и вычисление с примерами решенияDCA по 2-му признаку равенства треугольников (сторона AD — общая, углы при стороне AD соответственно равны по условию), то АВ = DC, Признаки равенства треугольников - определение и вычисление с примерами решенияB =Признаки равенства треугольников - определение и вычисление с примерами решенияC.

Так как Признаки равенства треугольников - определение и вычисление с примерами решенияBAO = Признаки равенства треугольников - определение и вычисление с примерами решенияBAD — Признаки равенства треугольников - определение и вычисление с примерами решенияCAD, Признаки равенства треугольников - определение и вычисление с примерами решенияCDO = Признаки равенства треугольников - определение и вычисление с примерами решенияCDA — Признаки равенства треугольников - определение и вычисление с примерами решенияBDA, тo Признаки равенства треугольников - определение и вычисление с примерами решенияBAO =Признаки равенства треугольников - определение и вычисление с примерами решенияCDO (если от равных углов отнять равные, получим равные углы). Тогда Признаки равенства треугольников - определение и вычисление с примерами решенияАОВ = Признаки равенства треугольников - определение и вычисление с примерами решенияDOC по 2-му признаку равенства треугольников.

Высота, медиана и биссектриса треугольника

У треугольника, помимо трех сторон, трех вершин и трех углов, имеются также и другие элементы — высота, медиана и биссектриса.
Признаки равенства треугольников - определение и вычисление с примерами решения

Определение. Высотой треугольника (рис. 118, а) называется перпендикуляр, опущенный из вершины треугольника на противоположную сторону или на ее продолжение (отрезок ВН).

Определение. Медианой треугольника (рис. 118, б) называется отрезок, который соединяет вершину треугольника с серединой противоположной стороны (отрезок ВМ).

Определение. Биссектрисой треугольника (рис. 118, в) называется отрезок биссектрисы угла треугольника, соединяющий вершину треугольника с точкой пересечения биссектрисы с противоположной стороной (отрезок ВК).

В равных треугольниках равны соответствующие высоты, медианы и биссектрисы.

Если треугольник не равнобедренный, то высота, медиана и биссектриса, проведенные из одной вершины треугольника, не совпадают (рис. 119).

Признаки равенства треугольников - определение и вычисление с примерами решения

Поскольку у треугольника три вершины, то у него и три высоты, три медианы, три биссектрисы. Позже мы докажем, что высоты треугольника (или их продолжения) пересекаются в одной точке. Это же касается медиан треугольника (рис. 120) и его биссектрис (рис. 121).

Признаки равенства треугольников - определение и вычисление с примерами решения

Если треугольник остроугольный (рис. 122, а), то точка пересечения его высот находится внутри треугольника ABC. Если треугольник тупоугольный или прямоугольный (рис. 122, б, в), то продолжения высот пересекаются соответственно вне треугольника или в вершине прямого угла.

Признаки равенства треугольников - определение и вычисление с примерами решения

Точки пересечения высот, биссектрис и медиан называются замечательными точками треугольника.

Геометрия 3D

Тетраэдром или треугольной пирамидой называется многогранник, у которого все четыре грани — треугольники. Любую его грань можно принять за основание, а противолежащую вершину — за вершину пирамиды. Если точка S — вершина, а треугольник ABC — основание пирамиды, то перпендикуляр SH к плоскости ABC является высотой тетраэдра (рис. 124).
Признаки равенства треугольников - определение и вычисление с примерами решения

Равнобедренный треугольник

Определение. Треугольник называется равнобедренным, если у него две стороны равны.

Равные стороны называются боковыми сторонами, третья сторона — основанием, вершина, противолежащая основанию, — вершиной равнобедренного треугольника.

Рассмотрим некоторые свойства равнобедренного треугольника и один из его признаков.

Теорема (о свойстве углов при основании). В равнобедренном треугольнике углы при основании равны.

Дано: Признаки равенства треугольников - определение и вычисление с примерами решения (рис. 126).

Признаки равенства треугольников - определение и вычисление с примерами решения

Доказать: Признаки равенства треугольников - определение и вычисление с примерами решения

Доказательство:

Проведем биссектрису ВК треугольника ABC. Треугольники АВК и СВК равны по двум сторонам и углу между ними: сторона ВК — общая, АВ = ВС по условию, углы АВК и СВК равны по определению биссектрисы. Из равенства этих треугольников следует, что Признаки равенства треугольников - определение и вычисление с примерами решения Теорема доказана.

Теорема (о свойстве биссектрисы равнобедренного треугольника).

В равнобедренном треугольнике биссектриса, проведенная к основанию, является его медианой и высотой.

Дано: Признаки равенства треугольников - определение и вычисление с примерами решения — биссектриса (рис. 127).

Признаки равенства треугольников - определение и вычисление с примерами решения

Доказать: ВК — медиана и высота.

Доказательство:

Треугольники АВК и СВК равны по двум сторонам и углу между ними (см. предыдущую теорему). Из равенства треугольников следует, что АК=КС и Признаки равенства треугольников - определение и вычисление с примерами решения1 =Признаки равенства треугольников - определение и вычисление с примерами решения2. Так как углы 1 и 2 смежные, то их сумма равна 180°, поэтому Признаки равенства треугольников - определение и вычисление с примерами решения Следовательно, ВК — медиана и высота. Теорема доказана.

Замечание. Поскольку из вершины треугольника можно провести только одну биссектрису, одну высоту и одну медиану, то теорему можно сформулировать так: «Биссектриса, высота и медиана равнобедренного треугольника, проведенные из вершины к основанию, совпадают». То есть если по условию задачи дана высота равнобедренного треугольника, проведенная к основанию, то согласно данной теореме она является биссектрисой и медианой. Аналогично, если дана медиана равнобедренного треугольника, проведенная к основанию, то она является высотой и биссектрисой.

Теорема (признак равнобедренного треугольника). Если в треугольнике два угла равны, то он равнобедренный.

Дано: Признаки равенства треугольников - определение и вычисление с примерами решения

Доказать:Признаки равенства треугольников - определение и вычисление с примерами решения

Доказательство:

Мысленно перевернем треугольник ABC обратной стороной (рис. 128) и наложим перевернутый треугольник на треугольник ABC так, чтобы их стороны АС совпали, угол С совпал с углом А, угол А совпал с углом С.

Признаки равенства треугольников - определение и вычисление с примерами решения

Тогда перевернутый треугольник совместится с данным, и сторона ВС совместится со стороной АВ. Следовательно, АВ = ВС, т. е. Признаки равенства треугольников - определение и вычисление с примерами решенияАВС — равнобедренный. Теорема доказана.

Доказанный признак равнобедренного треугольника является теоремой, обратной теореме о свойстве углов при основании равнобедренного треугольника (рис. 129).

Признаки равенства треугольников - определение и вычисление с примерами решения

Напомним, что любая теорема состоит из условия — того, что дано, и заключения — того, что нужно доказать. У теоремы, обратной данной, условием является заключение данной теоремы, а заключением — условие данной.

Пример №5

Доказать, что в равнобедренном треугольнике биссектрисы, проведенные к боковым сторонам, равны между собой.

Доказательство:

Пусть в Признаки равенства треугольников - определение и вычисление с примерами решенияАВС АВ =ВС, АК и СМ — биссектрисы (рис. 130). Нужно доказать, что АК = СМ. Рассмотрим Признаки равенства треугольников - определение и вычисление с примерами решенияАКВ и Признаки равенства треугольников - определение и вычисление с примерами решенияСМВ. У них Признаки равенства треугольников - определение и вычисление с примерами решенияB — общий, АВ = ВС по условию, Признаки равенства треугольников - определение и вычисление с примерами решенияBAK = Признаки равенства треугольников - определение и вычисление с примерами решенияBCM как половины равных углов А и С при основании равнобедренного треугольника. Тогда Признаки равенства треугольников - определение и вычисление с примерами решенияАКВ = Признаки равенства треугольников - определение и вычисление с примерами решенияСМВ по 2-му признаку равенства треугольников, откуда АК = СМ. Что и требовалось доказать.

Замечание. Вторым способом доказательства будет рассмотрениеПризнаки равенства треугольников - определение и вычисление с примерами решенияАКС иПризнаки равенства треугольников - определение и вычисление с примерами решенияСМА и доказательство их равенства.

Пример №6

Доказать, что перпендикуляр, проведенный из центра окружности к хорде, делит эту хорду пополам.

Доказательство:

Пусть О — центр окружности, АВ — хорда, ОН — перпендикуляр к хорде АВ (рис. 131).

Признаки равенства треугольников - определение и вычисление с примерами решения

Отрезки OA и ОВ равны как радиусы. Поэтому треугольник АОВ — равнобедренный, а ОН — его высота, проведенная к основанию. Мы знаем, что высота равнобедренного треугольника, проведенная к основанию, является и медианой. А медиана делит сторону треугольника пополам, то есть АН = НВ. Что и требовалось доказать.

Признаки равнобедренного треугольника

Вы уже знаете один признак равнобедренного треугольника: «Если в треугольнике два угла равны, то треугольник равнобедренный». Докажем еще три признака равнобедренного треугольника, связанных с его высотой, медианой и биссектрисой.

Теорема. Если в треугольнике высота является медианой, то треугольник равнобедренный.

Дано: ВН — высота и медиана Признаки равенства треугольников - определение и вычисление с примерами решенияАВС (рис. 136).

Признаки равенства треугольников - определение и вычисление с примерами решения

Доказать: АВ = ВС.

Доказательство:

Рассмотрим Признаки равенства треугольников - определение и вычисление с примерами решенияАВН и Признаки равенства треугольников - определение и вычисление с примерами решенияСВН. У них сторона ВН — общая, Признаки равенства треугольников - определение и вычисление с примерами решения Признаки равенства треугольников - определение и вычисление с примерами решения (так как ВН — высота), АН = СН (так как ВН — медиана). Треугольники АВН и СВН равны по двум сторонам и углу между ними. Из равенства треугольников следует равенство соответствующих сторон АВ и ВС. Теорема доказана.
 

Теорема. Если в треугольнике высота является биссектрисой, то треугольник равнобедренный.

Дано: ВН — высота и биссектриса Признаки равенства треугольников - определение и вычисление с примерами решенияАВС.

Доказать: АВ = ВС (рис. 137).

Признаки равенства треугольников - определение и вычисление с примерами решения

Доказательство:

Рассмотрим Признаки равенства треугольников - определение и вычисление с примерами решенияАВН и Признаки равенства треугольников - определение и вычисление с примерами решенияСВН. У них сторона ВН — общая, Признаки равенства треугольников - определение и вычисление с примерами решенияПризнаки равенства треугольников - определение и вычисление с примерами решения (так как ВН — высота), Признаки равенства треугольников - определение и вычисление с примерами решенияПризнаки равенства треугольников - определение и вычисление с примерами решения (так как ВН — биссектриса). Треугольники АВН и СВН равны по стороне и двум прилежащим к ней углам. Из равенства треугольников следует равенство соответствующих сторон АВ и ВС. Теорема доказана.

Теорема. Если в треугольнике медиана является биссектрисой, то треугольник равнобедренный.

Дано: ВМ — медиана и биссектриса Признаки равенства треугольников - определение и вычисление с примерами решенияАВС.

Доказать: АВ = ВС (рис. 138).

Доказательство:

Продлим медиану ВМ на ее длину за точку М. Получим МВХ = ВМ. Треугольники АМВ1 и СМВ равны по двум сторонам и углу между ними (МВ1 = ВМ по построению; AM = МС, так как ВМ — медиана; Признаки равенства треугольников - определение и вычисление с примерами решенияAMВ1 =Признаки равенства треугольников - определение и вычисление с примерами решенияCMB как вертикальные). Из равенства этих треугольников следует, что АВ1=ВС и Признаки равенства треугольников - определение и вычисление с примерами решенияAB1M = =Признаки равенства треугольников - определение и вычисление с примерами решенияCBM. Но ZCBM = ZABM, так как ВМ — биссектриса по условию. Тогда Признаки равенства треугольников - определение и вычисление с примерами решенияAB1B = Признаки равенства треугольников - определение и вычисление с примерами решенияABB1 и Признаки равенства треугольников - определение и вычисление с примерами решенияАВВ1 — равнобедренный по признаку равнобедренного треугольника. Следовательно, АВ=АВ1. А так как АВ1=ВС, то АВ = ВС. Теорема доказана.

Замечание. Прием продления (продолжения) медианы часто используется при решении геометрических задач.

Пример №7

В треугольнике ABC с периметром 54 см медиана АК перпендикулярна стороне ВС, а высота ВМ составляет равные углы со сторонами ВА и ВС. Найти стороны треугольника ABC.

Решение:

Так как медиана АК является и высотой, то Признаки равенства треугольников - определение и вычисление с примерами решенияАВС — равнобедренный с основанием ВС и АВ =АС. Так как высота ВМ является и биссектрисой, то Признаки равенства треугольников - определение и вычисление с примерами решенияАВС — равнобедренный с основанием АС и АВ = ВС. Тогда Признаки равенства треугольников - определение и вычисление с примерами решенияАВС — равносторонний, Признаки равенства треугольников - определение и вычисление с примерами решения Признаки равенства треугольников - определение и вычисление с примерами решения (см).

Ответ: 18 см.

Пример №8

Биссектриса АК треугольника АБС делит сторону ВС пополам. Периметр треугольника ABC равен 36 см, периметр треугольника АКС равен 30 см. Найти длину биссектрисы АК.

Решение:

Из условия следует, что биссектриса АК является и медианой Признаки равенства треугольников - определение и вычисление с примерами решенияАВС (рис. 139).

Признаки равенства треугольников - определение и вычисление с примерами решения

Тогда Признаки равенства треугольников - определение и вычисление с примерами решенияАВС — равнобедренный по признаку равнобедренного треугольника и АВ=АС. Так как ВК = СК, то сумма отрезков АС и СК равна полупериметру Признаки равенства треугольников - определение и вычисление с примерами решенияАВС, то есть 18 см. По условию периметр Признаки равенства треугольников - определение и вычисление с примерами решенияАКС равен 30 см, поэтому АК = 30 — 18 = 12 (см).

Ответ: 12 см

Геометрия 3D

У правильной треугольной пирамиды DABC в основании лежит равносторонний треугольник ABC, а боковые грани ADB, ADC, BDC — равные равнобедренные треугольники с общей вершиной D (рис. 142).

Признаки равенства треугольников - определение и вычисление с примерами решения

У правильной четырехугольной пирамиды в основании лежит квадрат MNKE, а боковые грани МРЕ, MPN, NPK, ЕРК — равные равнобедренные треугольники с общей вершиной Р (рис. 143).

Признаки равенства треугольников - определение и вычисление с примерами решения

Третий признак равенства треугольников

Вам уже известны два признака равенства треугольников. Рассмотрим еще один.

Теорема (третий признак равенства треугольников). Если три стороны одного треугольника соответственно равны трем сторонам другого треугольника, то такие треугольники равны.

Дано: АВ=А1В1, ВС = В1С1, АС=А1С1 (рис. 144).

Признаки равенства треугольников - определение и вычисление с примерами решения

Доказать: Признаки равенства треугольников - определение и вычисление с примерами решенияАВС = Признаки равенства треугольников - определение и вычисление с примерами решенияА1В1С1.

Доказательство:

Приложим треугольник А1В1С1 к треугольнику ABC так, чтобы у них совместились равные стороны А1С1 и АС, а вершины В1 и В оказались в разных полуплоскостях относительно прямой АС. Треугольник А1В1С1 займет положение треугольника АВ2С. Проведем отрезок ВВ2. Так как АВ2=АВ и В2С = ВС, то треугольники АВВ2 и СВВ2 — равнобедренные. Откуда Признаки равенства треугольников - определение и вычисление с примерами решенияl =Признаки равенства треугольников - определение и вычисление с примерами решения2 и Признаки равенства треугольников - определение и вычисление с примерами решения3 =Признаки равенства треугольников - определение и вычисление с примерами решения4 (как углы при основании равнобедренного треугольника). Тогда Признаки равенства треугольников - определение и вычисление с примерами решенияABC =Признаки равенства треугольников - определение и вычисление с примерами решенияAB2C, и треугольники ABC и АВ2С равны по двум сторонам и углу между ними. Следовательно, Признаки равенства треугольников - определение и вычисление с примерами решенияАВС =Признаки равенства треугольников - определение и вычисление с примерами решенияА1В1С1. Теорема доказана.

Замечание. Чтобы отрезок ВВ2 проходил внутри треугольника ABC, следует прикладывать треугольники большей стороной.

Говорят, что три стороны задают треугольник однозначно.

Итак, теперь вы знаете три признака равенства треугольников. Можно сформулировать и другие признаки равенства треугольников, в которых неизбежно будет присутствовать соответственное равенство каких-то трех элементов двух треугольников. Однако не любые три элемента задают треугольник. Так, например, если три угла одного треугольника соответственно равны трем углам другого треугольника, то такие треугольники не обязательно равны. То же касается треугольников, у которых соответственно равны две стороны и угол, противолежащий одной из этих сторон.

На рисунке 145, а, б вы видите пары таких неравных треугольников.

Признаки равенства треугольников - определение и вычисление с примерами решения

Пример №9

У простой замкнутой ломаной ABCD AB=AD, BC = DC. Доказать, что Признаки равенства треугольников - определение и вычисление с примерами решенияB = Признаки равенства треугольников - определение и вычисление с примерами решенияD и луч АС — биссектриса угла BAD.

Доказательство:

Проведем отрезок АС (рис. 146).

Признаки равенства треугольников - определение и вычисление с примерами решения

Треугольники ABC и ADC равны по 3-му признаку равенства треугольников (AB=AD и BC = DC по условию, сторона АС — общая). Поэтому Признаки равенства треугольников - определение и вычисление с примерами решенияB =Признаки равенства треугольников - определение и вычисление с примерами решенияD и Признаки равенства треугольников - определение и вычисление с примерами решенияBAC =Признаки равенства треугольников - определение и вычисление с примерами решенияDAC как соответствующие в двух равных треугольниках и луч АС — биссектриса угла BAD.

Пример №10

Доказать равенство треугольников по двум сторонам и медиане между ними.

Доказательство:

Пусть АВ =А1В1, ВС = В1С1, ВМ = В1М1, где ВМ и В1М1 — медианы (рис. 147).

Признаки равенства треугольников - определение и вычисление с примерами решения

Нужно доказать, что Признаки равенства треугольников - определение и вычисление с примерами решенияАВС =Признаки равенства треугольников - определение и вычисление с примерами решенияА1В1С1. Продлим в каждом треугольнике данную медиану на ее длину так, что MD = ВМ, M1D1=B1M1. Так как Признаки равенства треугольников - определение и вычисление с примерами решенияAMD =Признаки равенства треугольников - определение и вычисление с примерами решенияСМВ по 1-му признаку равенства треугольников (AM = МС, Признаки равенства треугольников - определение и вычисление с примерами решенияAMD =Признаки равенства треугольников - определение и вычисление с примерами решенияCMB как вертикальные, ВМ = MD по построению), то AD = BC. Аналогично Признаки равенства треугольников - определение и вычисление с примерами решенияAXMXDX = Признаки равенства треугольников - определение и вычисление с примерами решенияС1М1В1, откуда A1D1 = B1C1. По условию ВС = В1С1, следовательно, AD=A1D1 и Признаки равенства треугольников - определение и вычисление с примерами решенияABD =Признаки равенства треугольников - определение и вычисление с примерами решенияA1B1D1 по трем сторонам. Тогда Признаки равенства треугольников - определение и вычисление с примерами решенияABM =Признаки равенства треугольников - определение и вычисление с примерами решенияA1B1M1 и Признаки равенства треугольников - определение и вычисление с примерами решенияАВМ =Признаки равенства треугольников - определение и вычисление с примерами решенияА1В1М1 по 1-му признаку равенства треугольников. Отсюда AM =А1М1, АС =А1С1 (так как ВМ и В1М1 — медианы) и Признаки равенства треугольников - определение и вычисление с примерами решенияАВС =Признаки равенства треугольников - определение и вычисление с примерами решенияА1В1С1 по трем сторонам.

Пример №11

Два равных отрезка АВ и CD пересекаются в точке О и AD = BC. Доказать, что ВО = DO.

Доказательство:

Соединим точки В и D отрезком (рис. 148).

Признаки равенства треугольников - определение и вычисление с примерами решения

Треугольники ABD и CDB равны по трем сторонам (сторона BD — общая, AB=CD и AD=СВ по условию). Из равенства треугольников следует, что Признаки равенства треугольников - определение и вычисление с примерами решенияABD =Признаки равенства треугольников - определение и вычисление с примерами решенияCDB. Тогда Признаки равенства треугольников - определение и вычисление с примерами решенияBOD — равнобедренный (по признаку равнобедренного треугольника), откуда ВО=DO.

Серединный перпендикуляр к отрезку

Определение. Серединным перпендикуляром к отрезку называется прямая, перпендикулярная этому отрезку и проходящая через его середину.

Прямая CD — серединный перпендикуляр к отрезку АВ, то есть Признаки равенства треугольников - определение и вычисление с примерами решения(рис. 152).

Признаки равенства треугольников - определение и вычисление с примерами решения
Теорема (о серединном перпендикуляре).

Любая точка серединного перпендикуляра к отрезку равноудалена от концов этого отрезка. Если точка равноудалена от концов отрезка, то она лежит на серединном перпендикуляре к этому отрезку.

В данной теореме два утверждения: прямое и ему обратное. Докажем каждое из этих утверждений отдельно.

1) Дано: Признаки равенства треугольников - определение и вычисление с примерами решения — серединный перпендикуляр к отрезку Признаки равенства треугольников - определение и вычисление с примерами решения (рис. 153).

Признаки равенства треугольников - определение и вычисление с примерами решения

Доказать: КА = КВ.

Доказательство:

По определению серединного перпендикуляра Признаки равенства треугольников - определение и вычисление с примерами решения Тогда в треугольнике АКВ высота КМ является медианой. По признаку равнобедренного треугольника Признаки равенства треугольников - определение и вычисление с примерами решенияАКВ — равнобедренный, поэтому КА=КВ.

2) Дано: Признаки равенства треугольников - определение и вычисление с примерами решения (рис. 154).

Признаки равенства треугольников - определение и вычисление с примерами решения

Доказать: Признаки равенства треугольников - определение и вычисление с примерами решения где Признаки равенства треугольников - определение и вычисление с примерами решения — серединный перпендикуляр к отрезку АВ.

Доказательство:

Проведем в равнобедренном Признаки равенства треугольников - определение и вычисление с примерами решенияАКВ высоту КМ, которая по свойству равнобедренного треугольника будет и медианой. Получим Признаки равенства треугольников - определение и вычисление с примерами решения Прямая Признаки равенства треугольников - определение и вычисление с примерами решения, проходящая через высоту КМ, — серединный перпендикуляр к отрезку АВ.

Теорема доказана.

Геометрическим местом точек плоскости (или пространства) называется множество всех точек плоскости (или пространства), обладающих общим свойством.

Из доказанной теоремы следует, что серединный перпендикуляр к отрезку — это геометрическое место точек плоскости, равноудаленных от концов отрезка.

Пример №12

В четырехугольнике (рис. 155) ABCD  AB=BC, AD=DC.

Признаки равенства треугольников - определение и вычисление с примерами решения

Доказать, что  ACПризнаки равенства треугольников - определение и вычисление с примерами решенияBD.

Доказательство:

1-й способ. Из равенства треугольников ABD и CBD по трем сторонам следует, что Признаки равенства треугольников - определение и вычисление с примерами решенияABD =Признаки равенства треугольников - определение и вычисление с примерами решенияCBD. В равнобедренном треугольнике ABC биссектриса ВМ является и высотой. Поэтому  ACПризнаки равенства треугольников - определение и вычисление с примерами решенияBD.

2-й способ. Точки В и D равноудалены от концов отрезка АС, поэтому они лежат на серединном перпендикуляре к отрезку АС. Так как через две точки проходит единственная прямая, то BD — серединный перпендикуляр к отрезку АС. Отсюда ACПризнаки равенства треугольников - определение и вычисление с примерами решенияBD. и AM = МС.

Пример №13 (1-я замечательная точка треугольника).

Доказать, что серединные перпендикуляры к сторонам треугольника пересекаются в одной точке.

Доказательство:

Пусть два серединных перпендикуляра к сторонам АС и АВ пересекаются в точке О (рис. 156).

Признаки равенства треугольников - определение и вычисление с примерами решения

Точка О лежит на серединном перпендикуляре ОМ, поэтому ОА = ОС. Точка О лежит на серединном перпендикуляре ОК, поэтому ОА = ОВ. Отсюда ОВ = ОС. Поскольку точка О равноудалена от концов отрезка ВС, то она лежит на серединном перпендикуляре к отрезку ВС. Таким образом, третий серединный перпендикуляр пройдет через точку О, и все три серединных перпендикуляра к сторонам треугольника пересекутся в одной точке.

Замечания.

  • 1. Если ножку циркуля поставить в точку О и построить окружность радиусом OA, то она пройдет через все вершины треугольника в силу того, что OA = OB = ОС. Такая окружность называется описанной около треугольника. В данной задаче мы доказали, что центр окружности, описанной около треугольника, лежит в точке пересечения серединных перпендикуляров к его сторонам.
  • 2. Точка пересечения серединных перпендикуляров к сторонам треугольника — это еще одна замечательная точка треугольника помимо уже известных вам точек пересечения биссектрис, медиан, высот.

Напомню:

Три признака равенства треугольников:

  • По двум сторонам и углу между ними.
  • По стороне и двум прилежащим к ней углам.
  • По трем сторонам.

Запомните:

  1. Углы при основании равнобедренного треугольника равны.
  2. Биссектриса равнобедренного треугольника, проведенная из вершины к основанию, является его высотой и медианой.
  3. Если два угла треугольника равны, то треугольник равнобедренный (признак равнобедренного треугольника).
  4. Если высота треугольника является его медианой или биссектрисой, или медиана является его биссектрисой, то треугольник равнобедренный (признаки равнобедренного треугольника).
  5. Любая точка серединного перпендикуляра к отрезку равноудалена от концов этого отрезка. Если точка равноудалена от концов отрезка, то она лежит на серединном перпендикуляре к этому отрезку.
  6. Все три серединных перпендикуляра к сторонам треугольника пересекаются в одной точке (1-я замечательная точка треугольника).
  • Признаки равенства прямоугольных треугольников
  • Соотношения в прямоугольном треугольнике
  • Сумма углов треугольника
  • Внешний угол треугольника
  • Задачи на построение циркулем и линейкой
  • Задачи на построение по геометрии
  • Угол — определение, виды, как обозначают с примерами
  • Перпендикулярные прямые в геометрии
Автор статьи

Эксперт по предмету «Математика»

Задать вопрос автору статьи

Существует три признака равенства для двух треугольников. В этой статье мы рассмотрим их в виде теорем, а также приведем их доказательства. Для этого вспомним, что фигуры будут равны в том случае, когда они будут целиком накладываться друг на друга.

Первый признак

Два треугольника будут равными, если две стороны и угол между ними одного из треугольников будут равняться двум сторонам и углу, лежащему между ними в другом.

Доказательство.

Рассмотрим два треугольника $ABC$ и $A’B’C’$, в которых $AB=A’B’$,$AC=A’C’$ и $∠A=∠A’$ (рис. 1).

Совместим высоты $A$ и $A’$ этих треугольников. Так как углы при этих вершинах равны между собой, то стороны $AB$ и $AC$ наложатся, соответственно, на лучи $A’B’$ и $A’C’$. Так как эти стороны попарно равны, то стороны $AB$ и $AC$, соответственно, совпадут со сторонами $A’B’$ и $A’C’$, а следовательно и вершины $B$ и $B’$, $C$ и $C’$ будут совпадать.

Следовательно, сторона BC полностью совпадет со стороной $B’C’$. Значит, и треугольники будут целиком накладываться друг на друга, что и означает их равенства.

Теорема доказана.

Логотип baranka

Сдай на права пока
учишься в ВУЗе

Вся теория в удобном приложении. Выбери инструктора и начни заниматься!

Получить скидку 3 000 ₽

Второй признак

Теорема 2

Два треугольника будут равными, если два угла и их общая сторона одного из треугольников будут равняться двум углам и их общей стороны в другом.

Доказательство.

Рассмотрим два треугольника $ABC$ и $A’B’C’$, в которых $AC=A’C’$ и
$∠A=∠A’$, $∠C=∠C’$ (рис. 2).

Совместим стороны $AC$ и $A’C’$ этих треугольников, так что высоты $B$ и $B’$ будут лежать по одну сторону от нее. Так как углы при этих сторонах попарно равны между собой, то стороны $AB$ и $BC$ наложатся, соответственно, на лучи $A’B’$ и $B’C’$. Следовательно, и точка $B$ и точка $B’$ будет точками пересечения совмещенных лучей (то есть, к примеру, лучей $AB$ и $BC$). Так как лучи могут иметь только одну точку пересечения, то точка $B$ совпадет с точкой $B’$. Значит, и треугольники будут целиком накладываться друг на друга, что и означает их равенства.

Теорема доказана.

Третий признак

Теорема 3

Два треугольника будут равными, если три стороны одного из треугольников будут равняться трем сторонам в другом.

Доказательство.

Рассмотрим два треугольника $ABC$ и $A’B’C’$, в которых $AC=A’C’$,
$AB=A’B’$ и $BC=B’C’$ (рис. 3).

Доказательство.

Совместим стороны $AC$ и $A’C’$ этих треугольников, так что высоты $B$ и $B’$ будут лежать по разную сторону от нее. Далее будем рассматривать три различных случая полученного после этого расположения этих вершин. Будем их рассматривать на рисунках.

Первый случай:

Так как $AB=A’B’$, то будет верно равенство $∠ABB’=∠AB’B$. Аналогично, $∠BB’C=∠B’BC$. Тогда, как сумму, получим
$∠B=∠B’$

Следовательно, по теореме 1, эти треугольники равны.

Второй случай:

Так как $AB=A’B’$, то будет верно равенство $∠ABB’=∠AB’B$. Аналогично, $∠BB’C=∠B’BC$. Тогда, как разность, получим
$∠B=∠B’$

Следовательно, по теореме 1, эти треугольники равны.

Третий случай:

Так как $BC=B’C’$, то будет верно равенство $∠ABC=∠AB’C$

Следовательно, по теореме 1, эти треугольники равны.

Теорема доказана.

«Признаки равенства треугольников» 👇

Пример задач

Пример 1

Докажите равенство треугольников на рисунке ниже

Доказательство.

Углы $MKN$ и $PKE$ будут вертикальными. Следовательно, $∠MKN=∠PKE$. Значит, с учетом условия задачи, треугольники будут равны по теореме 2.

Пример 2

Докажите равенство треугольников на рисунке ниже

Доказательство.

Углы $AOB$ и $DOC$ будут вертикальными. Следовательно, $∠AOB=∠DOC$. Значит, с учетом условия задачи, треугольники будут равны по теореме 1.

Пример 3

Докажите равенство треугольников на рисунке ниже

Доказательство.

Так как сторона $FS$ является общей для этих треугольников, то, с учетом условия задачи, треугольники будут равны по теореме 1.

Находи статьи и создавай свой список литературы по ГОСТу

Поиск по теме

Третий признак равенства треугольников и его доказательство (всех трех возможных случаев) будут подробно рассмотрены в данной статье.

Формулировка третьего признака равенства треугольников

Если три стороны одного треугольника равны соответственно трем сторонам другого треугольника, то такие треугольники равны.

Доказательство

Дано:

2 треугольника, АВС и А1В1С1, AB = A1B1, AC = A1C1, BC = B1C1

Доказательства равных треугольников: как доказать равенство углов, 3 признака равенства, подобие треугольников

Требуется доказать, что треугольники АСВ и А1В1С1 равны.

Доказательство

Для начала необходимо «наложить» данные треугольники друг на друга таким образом – чтобы точка А совпала с точкой А1, точка В с точкой В1, а точки С и С1 оказались по разные стороны от прямой А1В1.

Доказательства равных треугольников: как доказать равенство углов, 3 признака равенства, подобие треугольников

Три возможных случая при наложении треугольников

  1. Луч С1С расположен внутри угла А1С1В1. Доказательства равных треугольников: как доказать равенство углов, 3 признака равенства, подобие треугольников
  2. Луч С1С накладывается на одну из сторон данного угла. Доказательства равных треугольников: как доказать равенство углов, 3 признака равенства, подобие треугольников
  3. Луч С1С расположен вне угла А1С1В1. Доказательства равных треугольников: как доказать равенство углов, 3 признака равенства, подобие треугольников

Доказательства равенства треугольников для трех возможных случаев

Первый случай

Луч С1С расположен внутри угла А1С1В1.

Доказательство:

  1. Рассмотрим треугольники В1С1С и АС1С. Доказательства равных треугольников: как доказать равенство углов, 3 признака равенства, подобие треугольников
  2. По условию стороны АС=А1С1, ВС=В1С1, следовательно, треугольники В1С1С и А1С1С – равнобедренные.
  3. Вспомнив, что углы при основании равнобедренных треугольников равны (свойство равнобедренного треугольника), получаем: ∠АСС1 = ∠А1С1С, ∠ВСС1 = ∠В1С1С.
  4. Поскольку ∠ACB = ∠ACC1 + ∠BCC1, ∠AC1B = ∠AC1C + ∠BC1C, то и углы AСB и AС1B равны.
  5. Так как ВС = В1С1, АС = А1С1 и ∠AСB = ∠AС1B, можно утверждать, что треугольники АВС и А1В1С1 равны согласно первому признаку равенства треугольников (по двум сторонам и углу между ними).

Что и требовалось доказать

Второй случай

Луч С1С накладывается на одну из сторон этого угла.

Доказательство:

  1. Рассмотрим треугольник САС1. Доказательства равных треугольников: как доказать равенство углов, 3 признака равенства, подобие треугольников
  2. Согласно условию теоремы, в треугольнике САС1 стороны АС и А1С1 равны, следовательно, сам треугольник САС1 — равнобедренный.
  3. По аналогии с доказательством первого случая (пункты 3-5): так как треугольник САС1 равнобедренный, то углы при его основании (СС1) равны, то есть ∠С = ∠С1 . Отсюда следует, что треугольники АВС и А1В1С1 равны по двум сторонам и углу между ними.

Что и требовалось доказать.

Третий случай

Луч С1С расположен вне угла А1С1В1.

Доказательство:

  1. Рассмотрим полученный треугольник ВСС1. Доказательства равных треугольников: как доказать равенство углов, 3 признака равенства, подобие треугольников
  2. По условию, стороны В1С1 и ВС – равны, следовательно, треугольник В1С1С – равнобедренный, а значит, что углы BСD и BС1D равны.
  3. Рассмотрим треугольник АСС1.
  4. Согласно условию, стороны АС и А1С1 – равны, отсюда следует, что треугольник АСС1 – равнобедренный и углы при его основании равны (∠DC1A = ∠DCA).
  5. ∠DCA = ∠DCB + ∠ACB, а ∠DC1A = ∠DC1B + ∠AC1B.
  6. Поскольку ∠DC1A = ∠DCA и ∠BСD = ∠BС1D, то отсюда следует, что и углы ∠АСВ и ∠АС1В равны.
  7. Исходя из вышенаписанного можно сделать вывод, что треугольники АВС и А1В1С1 равны по двум сторонам и углу между ними.

Что и требовалось доказать.

Источник: https://people-ask.ru/nauki/geometriya/tretij-priznak-ravenstva-treugolnikov-formulirovka-i-dokazatelstvo

Признаки подобия треугольников

Напомним для начала определение подобных треугольников.

Определение 1

Два треугольника называются подобными, если углы все углы одного треугольника соответственно равны углам другого и треугольника, и все сходственные стороны этих треугольников пропорциональны.

Для определения подобия треугольников существуют три признака подобия треугольников. Рассмотрим и докажем их.

Первый признак подобия треугольников

Теорема 1

Теорема 1: Если два угла одного треугольника соответственно равны двум углам второго треугольника, то такие треугольники подобны.

Доказательство.

Рассмотрим треугольники $ABC$ и $A_1B_1C_1$, у которых $angle A=angle A_1, angle B=angle B_1$. (рис. 1).

Доказательства равных треугольников: как доказать равенство углов, 3 признака равенства, подобие треугольников

  • Рисунок 1. Иллюстрация теоремы 1
  • Нам нужно доказать, что $angle C=angle C_1,$ и что $frac{AB}{A_1B_1}=frac{BC}{{B_1C}_1}=frac{AC}{A_1C_1}$.
  • По теореме о сумме углов треугольника, имеем:
  • Далее будем пользоваться следующей теоремой:

Теорема 2

Теорема 0: Если угол одного треугольника равен углу второго треугольника, то их площади относятся как произведения сторон, прилегающих к этому углу.

Доказательства равных треугольников: как доказать равенство углов, 3 признака равенства, подобие треугольников

Ничего непонятно?

Попробуй обратиться за помощью к преподавателям

  1. По теореме 0, получим
  2. Из этих равенств, получим
  3. Теорема доказана.

Второй признак подобия треугольников

Теорема 3

Теорема 2: Если две стороны одного треугольника пропорциональны соответствующим сторонам второго треугольника и углы между этими сторонами равны, то данные треугольники подобны.

Доказательство.

Рассмотрим треугольники $ABC$ и $A_1B_1C_1$, у которых $angle A=angle A_1$ и$frac{AB}{A_1B_1}=frac{AC}{A_1C_1}=k$ (рис. 2).

Доказательства равных треугольников: как доказать равенство углов, 3 признака равенства, подобие треугольников

Рисунок 2. Иллюстрация теоремы 2

Используя теорему 1, видим, что для доказательства этой теоремы, достаточно доказать, что $angle C=angle C_1$. Построим треугольник $ACB_2$, так, что $angle CAB_2=angle A_1$, а $angle B_2CA=angle C_1$ (рис. 2).

Доказательства равных треугольников: как доказать равенство углов, 3 признака равенства, подобие треугольников

Рисунок 3. Дополнительное построение

Треугольник $ACB_2$ подобен треугольнику $ABC$ (по теореме 1), следовательно,$ frac{AC}{A_1C_1}$ $=frac{AB_2}{A_1B_1}$. По условию $frac{AB}{A_1B_1}=frac{AC}{A_1C_1}$, следовательно, $AB=AB_2$. Тогда треугольник $ACB_2$ равен треугольнику $ABC$ по двум сторонам и углу между ними. Следовательно, $angle B_2CA=angle C$, а так как $angle B_2CA=angle C_1, то angle C=angle C_1.$

По первому признаку подобия треугольника получаем доказательство теоремы.

Третий признак подобия треугольников

Теорема 4

Теорема 3: Если три стороны одного треугольника пропорциональны трем соответствующим сторонам второго треугольника, то такие треугольники подобны.

Доказательство.

Рассмотрим треугольники $ABC$ и $A_1B_1C_1$, у которых $frac{AB}{A_1B_1}=frac{BC}{{B_1C}_1}=frac{AC}{A_1C_1}=k$.

Используя теорему 2, видим, что для доказательства этой теоремы, достаточно доказать, что $angle A=angle A_1$. Построим треугольник $ACB_2$, так, что $angle CAB_2=angle A_1$, а $angle B_2CA=angle C_1$ (рис. 3).

Доказательства равных треугольников: как доказать равенство углов, 3 признака равенства, подобие треугольников

Рисунок 4. Дополнительное построение

Треугольник $ACB_2$ подобен треугольнику $ABC$ (по теореме 1), следовательно,$ frac{AC}{A_1C_1}$ $=frac{AB_2}{A_1B_1}=frac{CB_2}{C_1B_1}$. Принимая во внимание равенства$frac{AB}{A_1B_1}=frac{BC}{{B_1C}_1}=frac{AC}{A_1C_1}$, получим, что $CB_2=CB, AB_2=AB$. Тогда треугольник $ACB_2$ равен треугольнику $ABC$ по трем сторонам. Следовательно, $angle A=angle A_1$.

Теорема доказана.

Пример задачи на использование признаков подобия

Пример 1

  • Доказать, что любые два равнобедренных треугольника, у которых углы между равными сторонами равны, являются подобными.
  • Решение.
  • Пусть даны равнобедренные треугольники $ABC$ и $A_1B_1C_1$ с $angle A=angle A_1.$ Так как треугольник $ABC$ равнобедренный, то
  • Так как треугольник $A_1B_1C_1$ равнобедренный, то

[angle B=angle C=frac{180-angle A}{2}]
[angle B_1=angle C_1=frac{180-A_1}{2}=frac{180-angle A}{2}=angle B=angle C]

То есть $angle B=angle B_1, angle C=angle C_1$. По теореме 1, получаем, что треугольники $ABC$ и $A_1B_1C_1$ подобны.

ч. т. д.

Источник: https://spravochnick.ru/matematika/podobnye_treugolniki/priznaki_podobiya_treugolnikov/

3 признак равенства треугольников

  • Теорема
  • (Третий признак равенства треугольников — по трём сторонам)
  • Если три стороны одного треугольника соответственно равны трём сторонам другого треугольника, то такие треугольники равны.

Доказательства равных треугольников: как доказать равенство углов, 3 признака равенства, подобие треугольников

  1. ΔABC,
  2. ΔA1B1C1,
  3. AB=A1B1, AC=A1C1, BC=B1C1.
  4. Доказать:
  5. ΔABC= ΔA1B1C1
  6. Доказательство:
  7. Приложим треугольник A1B1C1 к треугольнику ABC так, чтобы
  • вершина A1 совместилась с вершиной A,
  • вершина B1 совместилась с вершиной B,
  • точки C1 и C лежали по разные стороны от прямой AB.

При этом возможны три случая взаимного расположения луча CC1 и угла ACB.

I. Луч CC1 проходит внутри угла ACB.

  • Проведём отрезок CC1.
  • По условию AC=A1C1 и BC=B1C1, поэтому треугольники ACC1 и BCC1 — равнобедренные с основанием CC1.
  • По свойству равнобедренного треугольника, ∠ACC1=∠AC1C и ∠BCC1=∠BC1C.
  • Если к равным углам прибывать равные углы, то получим равные углы:

Доказательства равных треугольников: как доказать равенство углов, 3 признака равенства, подобие треугольников

  1. Таким образом, ∠ACB=∠AC1B.
  2. Точки A1 и A, B1 и B совмещены, то есть ∠AC1B и ∠A1C1B1 — один и тот же угол.
  3. Для треугольников ABC и A1B1C1 имеем:
  4. AC=A1C1, BC=B1C1 (по условию), ∠ACB=∠A1C1B1 (по доказанному).
  5. Следовательно, ΔABC= ΔA1B1C1 (по 1 признаку равенства треугольников).

Доказательства равных треугольников: как доказать равенство углов, 3 признака равенства, подобие треугольниковII. Луч CC1 проходит внутри угла ACB.

Так как AC=A1C1 и BC=B1C1, треугольники ACC1 и BCC1 — равнобедренные с основанием CC1 и ∠ACC1=∠AC1C и ∠BCC1=∠BC1C (как углы при основании).

Если из равных углов вычесть равные углы, то получим равные углы:

Доказательства равных треугольников: как доказать равенство углов, 3 признака равенства, подобие треугольников

Таким образом, ∠ACB=∠AC1B и ΔABC= ΔA1B1C1 (по 1 признаку равенства треугольников).

Доказательства равных треугольников: как доказать равенство углов, 3 признака равенства, подобие треугольниковIII. Луч CC1 совпадает со стороной угла ACB.

  • По условию BC=B1C1, поэтому треугольник BCC1 — равнобедренный с основанием CC1.
  • Отсюда ∠C1=∠C (как углы при основании) и ΔABC= ΔA1B1C1 (по 1 признаку равенства треугольников).
  • Что и требовалось доказать.

Источник: http://www.treugolniki.ru/tretij-priznak-ravenstva-treugolnikov/

math-public:priznaki-ravenstva-treugolnikov [Президентский ФМЛ №239]

math-public:priznaki-ravenstva-treugolnikov

Треугольник – это геометрическая фигура, которая состоит из трёх
точек, не лежащих на одной прямой, и трёх отрезков, соединяющих эти
три точки.

Треугольники называются равными, если их можно совместить наложением.

Пусть на прямой $AB$ точка $O$ лежит между точками $A$ и $B$. Если
от лучей $OA$ и $OB$ в разные полуплоскости отложить лучи $OC$ и
$OD$ соответственно так, чтобы $angle COA=angle DOB$, то точки $C,
O$ и $D$ лежат на одной прямой.

  • Предположим противное.
  • Тогда продолжим луч $CO$ за точку $O$: получим луч $OC_1$
  • Тогда $angle COA=angle BOC_1$, как вертикальные, и от луча $OB$ отложены два равных угла $angle DOB$ и $angle COC_1$, что противоречит аксиоме.

Если две стороны и угол между ними одного треугольника соответственно равны двум сторонам и углу между ними другого
треугольника, то такие треугольники равны.

  1. Рассмотрим треугольники $ABC$ и $A_1B_1C_1$, у которых $AB=A_1B_1,
    AC=A_1C_1, angle A=angle A_1$.
  2. Докажем, что $ riangle ABC= riangle A_1B_1C_1$.
  3. Так как $angle A=angle A_1$, то согласно аксиоме треугольник $ABC$ можно наложить на треугольник
    $A_1B_1C_1$ так, что вершина $A$ совместиться с вершиной $A_1$, а стороны $AB$ и $AC$ наложатся соответственно на лучи $A_1B_1$ и $A_1C_1$.
  4. В силу аксиомы, так как $AB=A_1B_1$ и $AC=A_1C_1$, то стороны $AB$ и $A_1B_1$, $AC$ и $A_1C_1$
    совместиться.
  5. В частности совместятся точки $B$ и $B_1$, $C$ и $C_1$.
  6. Следовательно, по аксиоме совместятся и стороны $BC$ и $B_1C_1$.
  7. Итак, треугольники $ABC$ и $A_1B_1C_1$ полностью совместились.
  8. Следовательно, согласно определению, они равны.

Если сторона и два прилежащих к ней угла одного треугольника
соответственно равны стороне и двум прилежащим к ней углам другого
треугольника, то такие треугольники равны.

  • Рассмотрим треугольники $ABC$ и $A_1B_1C_1$, у которых $AB=A_1B_1,
    angle A=angle A_1, angle B=angle B_1$.
  • Докажем, что $ riangle ABC= riangle A_1B_1C_1$.
  • Наложим треугольник $ABC$ на треугольник $A_1B_1C_1$ так, чтобы вершина $A$ совместилась с вершиной $A$,
    сторона $AB$ – с равной ей стороной $A_1B_1$, а вершины $C$ и $C_1$
    оказались по одну сторону от прямой $A_1B_1$.
  • Так как $angle A=angle A_1$ и $angle B=angle B_1$, то по сторона $AC$ наложится
    на луч $A_1C_1$, а сторона $BC$ – на луч $B_1C_1$.
  • Поэтому вершина $C$ – общая точка сторон $AC$ и $BC$ – окажется как лежащей на
    луче $A_1C_1$, так и на луче $B_1C_1$ и, следовательно, совместиться
    с общей точкой этих лучей – вершиной $C_1$.
  • Значит, совместятся стороны $AC$ и $A_1C_1$, $BC$ и $B_1C_1$.
  • Итак треугольники $ABC$ и $A_1B_1C_1$ полностью совместятся.
  • Следовательно, они равны.

Если три стороны одного треугольника соответственно равны трем
сторонам другого треугольника, то такие треугольники равны.

  1. Рассмотрим треугольники $ABC$ и $A_1B_1C_1$, у которых $AB=A_1B_1,
    AC=A_1C_1, BC=B_1C_1$.
  2. Докажем, что $ riangle ABC= riangle A_1B_1C_1$.
  3. Приложим треугольник $ABC$ к треугольнику $A_1B_1C_1$ так, чтобы
    вершина $A$ совместилась с вершиной $A_1$, вершина $B$ – C вершиной
    $B_1$, а вершины $C$ и $C_1$ оказались по разные стороны от прямой
    $A_1B_1$.
  4. Возможны три случая:
  1. луч $C_1C$ проходит внутри угла $A_1C_1B_1$

  2. луч $C_1C$ совпадает с одной из сторон этого угла

  3. луч $C_1C$ проходит вне угла $A_1C_1B_1$.

  • По условию теоремы $AB=A_1B_1, AC=A_1C_1, BC=B_1C_1$, следовательно, треугольники
    $A_1C_1C$ и $B_1C_1C$ – равнобедренные.
  • По теореме $angle 1=angle 2, angle 3=angle 4$, поэтому $angle
    A_1CB_1=angle A_1C_1B_1$.
  • Итак, $AC=A_1C_1, BC=B_1C_1, angle C=angle C_1$.
  • Следовательно, треугольники $ABC$ и $A_1B_1C_1$ равны по первому признаку равенства треугольников.
  1. Пусть луч $C_1C$ совпадает со стороной $C_1B$ угла $A_1C_1B_1$.
  2. Тогда, так как $AC=A_1C_1$, то треугольник $СA_1C_1$ равнобедренный, и ,следовательно, $angle C=angle C_1$.
  3. Тогда треугольники $A_1BC_1$ и $ABC$ равны по первому признаку равенства треугольников.

Третий случай доказывается аналогично первому.

math-public/priznaki-ravenstva-treugolnikov.txt · Последние изменения: 2016/05/06 11:15 — labreslav

Источник: http://wiki.sch239.net/math-public/priznaki-ravenstva-treugolnikov

Как доказать равенство треуголников? Примеры!

Надейка Высший разум (158170) 8 лет назад Первый признак равенства треугольников Если две стороны и угол между ними одного треугольника равны соответственно двум сторонам и углу между ними другого треугольника, то такие треугольники равны. Доказательство. Рассмотрим два треугольника ABC и A1B1C1.

Пусть в этих треугольниках равны стороны AB и A1B1, BC и B1C1, а угол ABC равен углу A1B1C1. Тогда треугольник A1B1C1 можно наложить на треугольник ABC так, чтобы угол A1B1C1 совпал с углом ABC. При этом можно расположить треугольник A1B1C1 так, чтобы сторона А1В1 совпала со стороной АВ, а сторона B1С1 — со стороной BС.

(В случае необходимости вместо треугольника A1B1C1 можно рассматривать равный ему «перевернутый» треугольник, т. е. треугольник, симметричный A1B1C1 относительно произвольной прямой .) Тогда треугольники совпадут полностью, поскольку совпадут все их вершины.

Второй признак равенства треугольников Если сторона и два прилежащих к ней угла одного треугольника равны соответственно стороне и двум прилежащим к ней углам другого треугольника, то такие треугольники равны. Доказательство. Пусть в треугольниках АВС и А 1 В 1 С 1 имеют место равенства AB= A1B1, ÐBAC = ÐB1A1C1, ÐАВС= ÐА1В1С1. Поступим так же, как и в предыдущем случае.

Наложим треугольник А1В1С1 на треугольник АВС так, чтобы совпали стороны AB и A1B1 и прилегающие к ним углы. Как и в предыдущем случае, при необходимости треугольник А1В1С1 можно «перевернуть обратной стороной». Тогда треугольники совпадут полностью. Значит, они равны.

Третий признак равенства треугольников Если три стороны одного треугольника равны соответственно трем сторонам другого треугольника, то такие треугольники равны. Доказательство. Пусть для треугольников ABC и A1B1C1 имеют место равенства АВ = А1В1, ВС = В1С1, СА = С1А1. Перенесем треугольник А1В1С1 так, чтобы сторона А1В1 совпала со стороной АВ, при этом должны совпасть вершины A1 и A, B1 и B.

Рассмотрим две окружности с центрами в A и B и радиусами соответственно AC и BC. Эти окружности пересекаются в двух симметричных относительно AB точках: C и C2. Значит, точка C1 после переноса указанным образом треугольника A1B1C1 должна совпасть либо с точкой C, либо с точкой C2.

В обоих случаях это будет означать равенство треугольников ABC и A1B1C1, поскольку треугольники ABC и ABC2 равны (эти треугольники симметричны относительно прямой AB.)

Источник: http://dcs. isa. ru/www /vladimirv /Geometry /dshar/sco_3.2.1/sco_3_2_1.html

Angelina11 Ученик (196) 2 года назад Геометрия, Признаки равенства треугольниковРис. 1Два треугольника называются равными, если их можно совместить наложением. На рисунке 1 изображены равные треугольники ABC и А1В1С1. Каждый из этих треугольников можно наложить на другой так, что они полностью совместятся, т. е. попарно совместятся их вершины и стороны. Ясно, что при этом совместятся попарно и углы этих треугольников.Таким образом, если два треугольника равны, то элементы (т. е. стороны и углы) одного треугольника соответственно равны элементам другого треугольника. Отметим, что в равных треугольниках против соответственно равных сторон (т. е. совмещающихся при наложении) лежат равные углы, и обратно: против соответственно равных углов лежат равные стороны.Так, например, в равных треугольниках ABC и A1B1C1, изображенных на рисунке 1, против соответственно равных сторон АВ и А1В1 лежат равные углы С и С1. Равенство треугольников ABC и А1В1С1 будем обозначать так: Δ ABC = Δ А1В1С1. Оказывается, что равенство двух треугольников можно установить, сравнивая некоторые их элементы.Признаки равенства треугольников, геометрия ЕГЭ и ГИАРис. 2Теорема 1. Первый признак равенства треугольников. Если две стороны и угол между ними одного треугольника соответственно равны двум сторонам и углу между ними другого треугольника, то такие треугольники равны (рис. 2).Доказательство. Рассмотрим треугольники ABC и A1B1C1, у которых АВ = A1B1, АС = A1C1 ∠ А = ∠ А1 (см. рис. 2). Докажем, что Δ ABC = Δ A1B1C1.Так как ∠ А = ∠ А1, то треугольник ABC можно наложить на треугольник А1В1С1 так, что вершина А совместится с вершиной А1, а стороны АВ и АС наложатся соответственно на лучи А1В1 и A1C1. Поскольку АВ = A1B1, АС = А1С1, то сторона АВ совместится со стороной А1В1 а сторона АС — со стороной А1C1; в частности, совместятся точки В и В1, С и C1. Следовательно, совместятся стороны ВС и В1С1. Итак, треугольники ABC и А1В1С1 полностью совместятся, значит, они равны.Аналогично методом наложения доказывается теорема 2.Признаки равенства треугольников, ГИА, ЕГЭРис. 3Теорема 2. Второй признак равенства треугольников. Если сторона и два прилежащих к ней угла одного треугольника соответственно равны стороне и двум прилежащим к ней углам другого треугольника, то такие треугольники равны (рис. 34).Замечание. На основе теоремы 2 устанавливается теорема 3.Теорема 3. Сумма любых двух внутренних углов треугольника меньше 180°.Из последней теоремы вытекает теорема 4.Теорема 4. Внешний угол треугольника больше любого внутреннего угла, не смежного с ним.

Теорема 5. Третий признак равенства треугольников. Если три стороны одного треугольника соответственно равны трем сторонам другого треугольника, то такие треугольники равны

Эдуард Голуб Профи (702) 2 года назад Геометрия, Признаки равенства треугольниковРис. 1Два треугольника называются равными, если их можно совместить наложением. На рисунке 1 изображены равные треугольники ABC и А1В1С1. Каждый из этих треугольников можно наложить на другой так, что они полностью совместятся, т. е. попарно совместятся их вершины и стороны. Ясно, что при этом совместятся попарно и углы этих треугольников.Таким образом, если два треугольника равны, то элементы (т. е. стороны и углы) одного треугольника соответственно равны элементам другого треугольника. Отметим, что в равных треугольниках против соответственно равных сторон (т. е. совмещающихся при наложении) лежат равные углы, и обратно: против соответственно равных углов лежат равные стороны.Так, например, в равных треугольниках ABC и A1B1C1, изображенных на рисунке 1, против соответственно равных сторон АВ и А1В1 лежат равные углы С и С1. Равенство треугольников ABC и А1В1С1 будем обозначать так: Δ ABC = Δ А1В1С1. Оказывается, что равенство двух треугольников можно установить, сравнивая некоторые их элементы.Признаки равенства треугольников, геометрия ЕГЭ и ГИАРис. 2Теорема 1. Первый признак равенства треугольников. Если две стороны и угол между ними одного треугольника соответственно равны двум сторонам и углу между ними другого треугольника, то такие треугольники равны (рис. 2).Доказательство. Рассмотрим треугольники ABC и A1B1C1, у которых АВ = A1B1, АС = A1C1 ∠ А = ∠ А1 (см. рис. 2). Докажем, что Δ ABC = Δ A1B1C1.Так как ∠ А = ∠ А1, то треугольник ABC можно наложить на треугольник А1В1С1 так, что вершина А совместится с вершиной А1, а стороны АВ и АС наложатся соответственно на лучи А1В1 и A1C1. Поскольку АВ = A1B1, АС = А1С1, то сторона АВ совместится со стороной А1В1 а сторона АС — со стороной А1C1; в частности, совместятся точки В и В1, С и C1. Следовательно, совместятся стороны ВС и В1С1. Итак, треугольники ABC и А1В1С1 полностью совместятся, значит, они равны.Аналогично методом наложения доказывается теорема 2.Признаки равенства треугольников, ГИА, ЕГЭРис. 3Теорема 2. Второй признак равенства треугольников. Если сторона и два прилежащих к ней угла одного треугольника соответственно равны стороне и двум прилежащим к ней углам другого треугольника, то такие треугольники равны (рис. 34).Замечание. На основе теоремы 2 устанавливается теорема 3.Теорема 3. Сумма любых двух внутренних углов треугольника меньше 180°.Из последней теоремы вытекает теорема 4.Теорема 4. Внешний угол треугольника больше любого внутреннего угла, не смежного с ним.

Теорема 5. Третий признак равенства треугольников. Если три стороны одного треугольника соответственно равны трем сторонам другого треугольника, то такие треугольники равны

Источник: https://otvet.mail.ru/question/48904505

«Нестандартные признаки равенства треугольников»

МБУ ДО города Ростова-на-Дону «Дворец творчества детей и молодежи»

Донская академия наук юных исследователей им. Ю. А. Жданова

  • Математика
  • Тема: «Нестандартные теоремы о равенстве треугольников»
  • Автор работы:
  • Подгорный Максим, 7 кл.,
  • МБОУ СОШ № 3,
  • г. Сальск, Ростовская область
  • Руководитель:
  • Олейникова Людмила Александровна,
  • учитель математики,
  • МБОУ СОШ № 3,
  • г. Сальск, Ростовская область
  • г. Ростов-на-Дону
  • 2017 год
  • Содержание
  • Введение………………………………………………………….………………3
  • Основная часть
  • Признаки равенства треугольников…………………………………………… 4
  • Нестандартные признаки равенства треугольников………………………….7
  • Заключение…………………………………………………………………… 10
  • Список литературы…………………………………………………………… 11
  • Приложение
  • Введение.
  • Актуальность:

Треугольник одна из основных фигур в планиметрии. Я много слышал от старшеклассников, что при подготовке к ЕГЭ им часто приходится доказывать равенство треугольников. И оказывается недостаточным знание основных признаков.

Мне захотелось узнать, а можно ли доказать равенство треугольников по другим параметрам . В учебнике геометрии, по которому обучаются ученики нашей школы ( авторы Л.С.Атанасян, В.Ф.Бутузов и др. Геометрия 7-9) рассматриваются всего 3 признака равенства треугольников.

Я просмотрел учебно-методические комплекты других авторов. Но и в них для изучения предлагаются только три известные теоремы.

Гипотеза:

Возможно, ли сформулировать, кроме трех известных, другие признаки равенства треугольников?

Чтобы убедиться в том, что ответ на этот вопрос волнует не только меня, я провел социологический опрос среди учащихся 7-11 классов      см. приложение  1 ).

Мои предположения подтвердились. Большенство учеников знают только  три признака равенства треугольников.

  1. Таким образом,  целью моего исследования стало отыскание новых признаков равенства треугольников.
  2. Задачи :
  3.        ΘИзучить литературу по исследуемой  теме.
  4.        ΘУточнить количество признаков равенства треугольников.
  5.        ΘПродемонстрировать своим одноклассникам и учащимся нашей  школы существование других признаков равенства треугольников  и возможности их доказательства.
  6. Объект исследования :
  7. Изучение признаков равенства  треугольников.

Предмет  исследования.  Треугольник, как одна из основных фигур в планиметрии.

  • Метод  исследования:  Теоретический ( изучение, анализ и синтез),системно-поисковый, практический (доказательство теорем ).
  • Историческая справка
  • Треугольник является одной из центральных фигур всей геометрии.
  • При решении задач используют его самые разнообразные свойства.
  • Свойства треугольника широко применяют на практике: в архитектуре; при разработке чертежа здания, при планировке будущих квартир; в промышленности, при проектировании различных деталей, при изготовлении стройматериалов, при строительстве морских и авиа судов; в навигации для проложения правильного и максимально точного маршрута; в астрологии и астрономии треугольник является очень значимой фигурой; треугольники делают надежными конструкции высоковольтных линий электропередач и железнодорожных мостов.

Кроме того, много других сфер, где применяются различные свойства треугольника: начиная игру в бильярд, необходимо расположить шары в виде треугольника, для этого используют специальное приспособление; расстановка кеглей в игре Боулинг тоже в виде равностороннего треугольника; для составления красивых паркетов используются треугольники; устройство треугольника Паскаля: каждое число равно сумме двух расположенных над ним чисел (обвести треугольником три числа). Все элементарно, но сколько в этом таится чудес! Треугольник Паскаля компьютер перевёл на язык цвета.

Тему треугольника можно продолжать неограниченно.

Каких только треугольников нет на свете!

Существуют также переносные значения данной фигуры: например, правило «золотого треугольника» основано на психологии покупателя – найдя нужный ему товар, покупатель устремляется в кассу. Задача продавцов – заставить его задержаться в магазине подольше, расположив нужный покупателю товар в вершинах воображаемого треугольника, то есть «заякорить» покупателя.

Чем больше площадь треугольника, тем более удачным можно назвать планировку магазина. В продуктовом магазине этими товарами-якорями являются гастрономия, молочная продукция, хлеб.

Задняя торцевая стена торгового зала является вторым местом по значимости и именно там целесообразнее всего располагать товары-якоря – именно для того, что бы заставить покупателя пройти весь периметр магазина.

Широко известный Бермудский треугольник – это район в Атлантическом океане, в котором происходят якобы таинственные исчезновения морских и воздушных судов. Район ограничен линиями от Флориды к Бермудским островам, далее к Пуэрто-Рико и назад к Флориде через Багамы.

  1. Поэтому изучение треугольника и всех его свойств – очень актуальная тема.
  2. Цель данной работы – рассказать о признаках равенства треугольников, что является одним из важнейших их свойств.
  3. Признаки равенства треугольников — это теоремы, на основании которых можно доказать, что некоторые треугольники равны.
  4. В геометрии используются три признака равенства треугольников.
  5. Данная тема практически изучена, так как на сегодняшний день существуют три признака равенства треугольников, доказываемых с помощью соответствующих теорем. 

В глубокой древности вместе с астрономией появилась наука – тригонометрия. Слово «тригонометрия» произведено от греческих «треугольник» и «меряю». Буквальное значение – «наука об измерении треугольников».

С помощью натянутых веревок длиной 3, 4 и 5 единиц египетские жрецы получали прямые углы при возведении храмов и т.п.

Искусство изображать предметы на плоскости с Древних времён привлекает к себе внимание человека, люди рисовали на скалах, стенах, сосудах и прочих предметах быта, различные орнаменты, растения, животных. Люди стремились к тому, чтобы изображение правильно отображало естественную форму предмета.

Учение о подобии фигур на основе теории отношений и пропорций было создано в Древней Греции в 5-4 веках до нашей эры и существует и развивается до сих пор.

Например, очень много детских игрушек подобным предметам взрослого мира, обувь и одежда одного фасона выпускается различных размеров. Эти примеры можно продолжать и дальше.

В конце концов, все люди подобны друг другу и как утверждает Библия, создал их бог по своему образу и подобию.

Признаки равенства треугольников имели издавна важнейшее значение в геометрии, так как доказательства многочисленных теорем сводилось к доказательству равенства тех или иных треугольников.

Доказательством признаков равенства треугольников занимались еще пифагорейцы.

По словам Прокла, Евдем Родосский приписывает Фалесу Милетскому доказательство о равенстве двух треугольников, имеющих равными сторону и два прилежащих к ней угла (второй признак равенства треугольников).

Эту теорему Фалес использовал для определения расстояния от берега до морских кораблей. Каким способом пользовался при этом Фалес, точно не известно.

Признаки  равенства  треугольников.

Начнем с определения. Треугольники  АВС  и  А1В1С1  называются равными, если их можно совместить наложением.

  • Треугольник состоит из  шести элементов: трех углов и трех сторон.
  • При этом возникает вопрос :   » Какое наименьшее количество элементов треугольника нужно взять для установления равенства   двух  треугольников ?»
  • Мы не сможем установить равенство двух треугольников по одному  элементу, потому что неизвестно :»Будут ли равны остальные элементы ?»

Источник: https://nsportal.ru/ap/library/nauchno-tekhnicheskoe-tvorchestvo/2017/08/06/nestandartnye-priznaki-ravenstva

Теоретические материалы: Соизмеримые и несоизмеримые отрезки. Отношение отрезков. Пропорциональные отрезки

Планиметрия

6. Подобие фигур

6.1. Соизмеримые и несоизмеримые отрезки. Отношение отрезков. Пропорциональные отрезки

Пусть некоторый отрезок содержится в отрезке ровно и в отрезке ровно раза: , . Отрезок в таком случае называют общей мерой отрезков и . Ясно, что если взять любую долю отрезка , то она также будет содержаться в каждом из данных отрезков и целое число раз. Например, если , то , ; если , то , и т. д.

Определение 1

Общей мерой двух отрезков будет называться такой третий отрезок, который содержится целое число раз в каждом из двух данных отрезков.

Если отрезки имеют общую меру, то они имеют бесконечное множество общих мер. Одна из них больше всех остальных и называется наибольшей общей мерой данных отрезков. Если меньший из двух данных отрезков содержится в большем целое число раз, то меньший отрезок и является наибольшей общей мерой двух данных отрезков.

Определение 2

Два отрезка ( и ) называются соизмеримыми, если они имеют общую меру (): и , где и — натуральные числа.

Соизмеримые отрезки существуют, например, м и дм. Для нахождения их наибольшей общей меры отложим на ( раза) и получим в остатке отрезок ( дм), меньший отрезка . Остаток отложится на меньшем отрезке ровно раза, т. е. и в большем отрезке раз, т. е.

, поэтому и есть наибольшая общая мера отрезков и . Если бы, откладывая остаток на меньшем данном отрезке , опять получили бы остаток (), то откладывали бы на и т.д., пока не получилось бы, что отложится в целое число раз без остатка.

Тогда и будет наибольшей общей мерой отрезков и .

Определение 3

Два отрезка называются несоизмеримыми, если они не имеют общей меры.

Примером несоизмеримых отрезков могут служить сторона и диагональ квадрата (доказательство этого факта опускаем) . Это значит, что процесс откладывания стороны квадрата на диагонали, остатка на стороне, второго остатка на первом и т. д. был бы бесконечен, так как всегда получался бы остаток.

Измерить отрезок единичным отрезком значит найти число, показывающее, сколько раз отложится на отрезке отрезок или его доли. Это число называют длиной отрезка. Если отрезок соизмерим с единичным отрезком , то длина отрезка есть число рациональное (натуральное или рациональная дробь). Например, дм при дм или м при м.

Если отрезок несоизмерим с единичным отрезком , то длина отрезка есть число иррациональное. Например, если дм сделать стороной квадрата, то его диагональ дм (по теореме Пифагора ).

Поскольку иррациональное число есть бесконечная непериодическая десятичная дробь, то его приближенное значение можно выразить с любой степенью точности с помощью конечной (рациональной) дроби. Так, точная длина есть дм, а приближенная дм или дм и т. д.

Определение 4

Отношением двух отрезков называется отношение их длин, выраженных в одинаковых единицах измерения.

Величина отношения отрезков не изменится, если в качестве единицы измерения взять любую общую меру данных отрезков. Так, если и , то и .

Определение 5

Четыре отрезка , , , называются пропорциональными, если из их длин можно составить пропорцию .

Этому определению не противоречит следующее более общее определение. Отрезки называются пропорциональными отрезкам , если .

Источник: https://dl.bsu.by/mod/book/view.php?id=10185&chapterid=1321

Первый признак равенства треугольников – доказательство: второй и третий признаки, теорема и определение

С далеких времен и по сей день поиск признаков равенства фигур считается базовой задачей, которая является основой основ геометрии; сотни теорем доказываются с использованием признаков равенства. Умение доказывать равенство и подобие фигур — важная задача во всех сферах строительства.

Применение навыка на практике

Предположим, что у нас есть фигура, начерченная на листе бумаги. При этом у нас есть линейка и транспортир, с помощью которых мы можем замерять длины отрезков и углы между ними. Как перенести на второй лист бумаги фигуру таких же размеров или увеличить ее масштаб в два раза.

Мы знаем, что треугольник — это фигура, состоящая из трех отрезков, называемых сторонами, образующими углы. Таким образом, существует шесть параметров — три стороны и три угла, которые определяют эту фигуру.

Однако, замерив величину всех трех сторон и углов, перенести данную фигуру на другую поверхность окажется непростой задачей. Кроме того, есть смысл задать вопрос: а не достаточно ли будет знания параметров двух сторон и одного угла, или всего лишь трех сторон.

Замерив длину двух сторон и угол между ними, затем отложим этот угол на новом листке бумаги, так мы сможем полностью воссоздать треугольник. Давайте разберемся, как это сделать, научимся доказывать признаки, по которым их можно считать одинаковыми, и определимся с тем, какое минимальное число параметров достаточно знать, чтобы получить уверенность в том, что треугольники одинаковы.

Важно ! Фигуры называются одинаковыми, если отрезки, образующие их стороны, и углы равны между собой. Подобными называются те фигуры, у которых стороны и углы пропорциональны. Таким образом, равенство — это подобие с коэффициентом пропорциональности 1.

Какие существуют признаки равенства треугольников, дадим их определение:

  • первый признак равенства: два треугольника можно считать одинаковыми, если равны две их стороны, а также угол между ними.
  • второй признак равенства треугольников: два треугольника будут одинаковыми, если одинаковы два угла, а также соответствующая сторона между ними.
  • третий признак равенства треугольников: треугольники можно считать одинаковыми, когда все их стороны имеют равную длину.

Как доказать, что треугольники равны. Приведем доказательство равенства треугольников.

Доказательство 1 признака

Долгое время среди первых математиков данный признак считался аксиомой, однако, как оказалось, его можно геометрически доказать, опираясь на более базовые аксиомы.

Рассмотрим два треугольника — KMN и K1M1N1. Сторона КМ имеет такую же длину как и K1M1, а KN = K1N1. А угол MKN равен углам KMN и M1K1N1.

Если рассматривать KM и K1M1, KN и K1N1 как два луча, которые выходят из одной точки, то можно сказать, что между этими парами лучей одинаковые углы (это задано условием теоремы). Произведем параллельный перенос лучей K1M1 и K1N1 из точки K1 в точку К. Вследствие этого переноса лучи K1M1 и K1N1 полностью совпадут.

Отложим на луче K1M1 отрезок длиной КМ, берущий свое начало в точке К. Поскольку по условию полученный отрезок и будет равен отрезку K1M1 то точки М и M1 совпадают. Аналогично и с отрезками KN и K1N1.

Таким образом, перенося K1M1N1 так, что точки K1 и К совпадают, а две стороны накладываются, получаем полное совпадение и самих фигур.

Важно! В интернете встречаются доказательства равенства треугольников по двум сторонам и углу при помощи алгебраических и тригонометрических тождеств с численными значениями сторон и углов. Однако исторически и математически данная теорема была сформулирована задолго до алгебры и раньше, чем тригонометрия. Для доказательства этого признака теоремы использовать что-либо, кроме базовых аксиом, некорректно.

Доказательство 2 признака

Докажем второй признак равенства по двум углам и стороне, основываясь на первом.

Доказательство 2 признака

Рассмотрим KMN и PRS. К равен Р, N равен S. Сторона КN имеет такую же длину, как и РS. Необходимо доказать, что KMN и PRS — одинаковы.

Отразим точку М относительно луча КN. Полученную точку назовем L. При этом длина стороны КМ = КL. NKL равен PRS. KNL равен RSP.

Поскольку сумма углов равна 180 градусов, то KLN равен PRS, а значит PRS и KLN- одинаковые (подобные) по обеим сторонам и углу, согласно первому признаку.

Но, так как KNL равен KMN, то KMN и PRS — две одинаковые фигуры.

Это интересно! Чему равна и как найти площадь равностороннего треугольника

Доказательство 3 признака

Как установить, что треугольники равны. Это прямо вытекает из доказательства второго признака.

Длина KN = PS. Поскольку К = Р, N = S, KL=KM, при этом КN = KS, MN=ML, то:

Это означает, что обе фигуры являются подобными друг другу. Но так как их стороны одинаковы, то и они также равны.

Из признаков равенства и подобия вытекает множество следствий. Одно из них заключается в том, что для того, чтобы определить, равны два треугольника или нет, необходимо знать их свойства, одинаковы ли:

  • все три стороны;
  • обе стороны и угол между ними;
  • оба угла и сторона между ними.

Использование признака равенства треугольников для решения задач

Это интересно! Изучаем математику в игровой форме: как ребенку быстро выучить таблицу умножения

Следствия первого признака

В ходе доказательства можно прийти к ряду интересных и полезных следствий.

  1. Параллелограмм. Тот факт, что точка пересечения диагоналей параллелограмма делит их на две одинаковые части — следствие признаков равенства и вполне поддается доказательству.Стороны дополнительного треугольника (при зеркальном построении, как в доказательствах, которые мы выполняли) — параллельны сторонам главного (стороны параллелограмма).
  2. Если есть два прямоугольных треугольника, у которых одинаковые острые углы, то они подобны. Если при этом катет первого равен катету второго, то они равны. Понять это довольно легко — у любых прямоугольных треугольников есть прямой угол. Поэтому признаки равенства для них более просты.
  3. Два треугольника с прямыми углами, у которых два катета имеют одинаковую длину, можно считать одинаковыми. Это связано с тем, что между двумя катетами угол всегда равен 90 градусов. Поэтому по первому признаку (по двум сторонам и углу между ними) все треугольники с прямыми углами и одинаковыми катетами — равны.
  4. Если есть два прямоугольных треугольника, и у них один катет и гипотенуза равны, значит и треугольники одинаковы.

Докажем эту простую теорему.

Есть два прямоугольных треугольника. У одного стороны a, b, c, где с — гипотенуза; a, b — катеты. У второго стороны n, m, l, где l — гипотенуза; m, n — катеты.

  • По теореме Пифагора один из катетов равен:
  • ;
  • .

Таким образом, если n = a, l = с (равенство катетов и гипотенуз), соответственно и вторые катеты будут равны. Фигуры, соответственно, будут равны по третьему признаку (по трем сторонам).

Отметим еще одно важное следствие. Если есть два равных треугольника, и они подобны с коэффициентом подобия k, то есть попарные отношения всех их сторон равны k, то отношение их площадей равно k2 .

Первый признак равенства треугольников. Видеоурок по геометрии 7 класс

Геометрия 7 Первый признак равенства треугольников

Вывод

Рассмотренная нами тема поможет любому ученику лучше разобраться в базовых геометрических понятиях и повысить свои навыки в интереснейшем мире математики.

Источник: https://uchim.guru/matematika/pervyj-priznak-ravenstva-treugolnikov-dokazatelstvo.html

Признаки подобия треугольников

Признаками подобия двух треугольников являются такие геометрические признаки, которые позволяют установить, что два неких треугольника являются подобными друг другу, без рассмотрения всех элементов.

Теорема 1

Первый признак подобия двух треугольников

 Треугольники подобны, если  хотя бы два угла в неком треугольнике соответственно равны двум углам в  другом треугольнике.

Доказательство

Если даны два треугольника: ABC и А1В1С1, где ∠A=∠A1 , и ∠B=∠B1. Тогда получается, что ∠C и ∠C1  также равны между собой. Давайте докажем, подобие △ABC и △A1B1C1.

Если отложить на стороне ВА отрезок ВА2, который будет равен отрезку A1B1 , и затем, провести прямую через точку А2, которая будет параллельна прямой АС. То эта прямая будет пресекать  отрезок ВС в точке, которую назовем С2 .

Итак, треугольники А2ВС2 и А1В1С1   равны: А2В =А1В1 по построению, ∠В1 = ∠В по условию и ∠А2= ∠А1 , так как ∠А=∠А1 по условию и ∠А2 =∠А как соответственные углы.

Согласно  лемме 1 о  подобных треугольниках (прямая, которая  параллельна одной из сторон треугольника и которая пересекает  две другие его стороны, отсекает  треугольник,  который подобен данному) будем иметь: △ABC ∼ △A2BC2 , таким образом, △A1B1C1 ∼△ABC. Значит, теорема доказана. Теоремы 2 и 3 доказываются по аналогичной схеме.

Теорема 2

Второй признак подобия треугольников.

Треугольники считаются подобными, если  две  из сторон одного треугольника будут соответственно пропорциональными двум сторонам второго треугольника. Также должно соблюдаться условие равенства углов между этими сторонами.

Теорема 3

Третий признак подобия треугольников.

Треугольники считаются подобными, если соблюдается условие пропорциональности  трех сторон одного из них трем сторонам второго.

Следствие 1 из теоремы 1. Если рассматривать подобные треугольники, то  их сходственные стороны будут пропорциональны высотам, которые будут опущены на сходственные стороны.

Признаки подобия прямоугольных треугольников

  1. прямоугольные треугольники считаются подобными, если катет и гипотенуза одного из них пропорциональны катету и гипотенузе второго треугольника;
  2. подобными считаются прямоугольные треугольники, если острый угол одного из них равен острому углу второго треугольника.

Признаки подобия треугольников в примерах

Пример 1

Необходимо найти длину отрезка KP, если известно, что в треугольнике АВС,  длина стороны АС равна десяти, и на стороне АВ есть некая точка К, но АК =2, ВК=3. Через точку К проведена прямая, которая параллельна АС. Точка P лежит на ее пересечении со стороной ВС. Это ситуация, когда используются  признаки подобия треугольников.

Урок с подобной задачкой обязательно встречается в каждой школе. Итак, если  в треугольнике есть прямая, проведенная параллельно одной стороне, то образуется треугольник, который  подобен данному. Треугольник КBР подобен треугольнику АBС. Доказывая это, заметим, что угол ВКР равен углу ВАС.

В виду того, что это соответственные углы, которые лежат при параллельных КР и АС и секущей АК. Кроме этого, угол В — общий и, следовательно, третьи углы равны, угол ВРК и ВСА. Таким образом, согласно теореме о  первом признаке подобия треугольников, ∠ АВС подобен ∠КВР.

Из этого следует, что КР / АС, стороны лежащие против ∠В, равно ВК / ВА стороны, стороны, которые лежат против равных ∠Р и ∠С. Следовательно, отрезок ВА найдем, складывая BК и АК. Подставляем сюда данные, получаем:  КР / 10 = 3 / 5  то есть, КР=6

Пример 2

Пусть в треугольниках ABC и А1В1С1, ∠В = ∠В1. Стороны АВ, ВС в треугольнике ABC больше в 2,5 раза сторон A1B1, B1C1, что в  треугольнике A1B1C1. Нужно найти АС и A1C1 , при условии, что их сумма равняется 4,2 м. Решение. По условию задачи запишем:

  1. ∠B=∠B1;
  2. AB/A1B1=BC/B1C1=2,5 Следовательно, △ABC∼△А1В1С1. По второму признаку подобия треугольников.
  3. AC+A1C1=4,2м. Из подобия этих треугольников получаем следствие AC/A1C1=2,5 , или АС=2,5xА1С1 Если АС = 2,5 x А1С1, то АС + А1C1 = 2,5 x А1С1 + A1C1 = 4,2, поэтому АС = 3 (м), A1C1 = 1,2 (м).

Пример 3

Необходимо выяснить, подобны ли треугольники А1В1С1 и  ABC если см, ВС = 5 см, АВ = 3, АС = 7 см, B1C1 = 7,5 см, А1В1 = 4,5 см, A1C1 = 10,5 см? Решение. ВС/ B1C1=5/7.5= 1/1.5 AB/ А1В1=3/4.5=1/1.5 АС/ A1C1=7/10.5=1/1.5

Значит, по третьему признаку, треугольники являются подобными. 

Источник: https://elhow.ru/ucheba/geometrija/planimetrija/priznaki-podobija-treugolnikov

Понравилась статья? Поделить с друзьями:
  • Как найти угол между прямыми в параллелограмме
  • Как составить объяснение значения слова
  • Как найти касательную в точке через производную
  • Как мне исправить двойку по математике
  • Как найти где ставить тире